Princeton NCLEX review

Pataasin ang iyong marka sa homework at exams ngayon gamit ang Quizwiz!

safety and infection control

----------

The nurse is caring for clients on the medical unit when the "Code Red" alert is announced over the intercom. The unit is not close to the fire's point of origin. Which of the following actions is appropriate? Close the double doors to the unit Assist all clients to the far end of the unit Use the elevators to begin the evacuation Relocate clients to a unit on a higher floor

Close the double doors to the unit Rationale: Upon activation of a code red, nurses who are not in proximity to the fire should first close all doors and keep patients and visitors in their rooms with the doors closed. Elevators may be unsafe, and clients would be evacuated using the stairwell. Clients are not relocated to higher floors during fires. Evacuation will occur horizontally, and then laterally, if there is immediate danger due to fire, smoke, chemical release, structural failure, or a similar condition.

The nurse notices flames and smoke in the garbage can in a client's room. Which action should the nurse take first? Extinguish the fire. Remove the client from the area. Close the door to the room. Activate the alarm system.

Remove the client from the area. Rationale: The nurse's first action in an active fire should be to remove the client from imminent harm. The other actions should occur after the client is taken to safety.

A nurse is preparing to move a client up in bed. The client is unable to assist in repositioning and assistance is obtained. Which action demonstrates use of ergonomic principles? Twist at the waist while lifting the client Raise the bed to the nurse's working height Keep the feet close together Lift and move in an uncoordinated fashion

Raise the bed to the nurse's working height Rationale: Twisting at the waist, keeping the feet close together when lifting and moving, and performing uncoordinated lifting and moving all increase the risk of back injuries in nurses. Raising the bed height reduces lower back strain.

A home health nurse is working in a neighborhood with a high crime rate. What procedures should the nurse follow to reduce safety risks? Have local law enforcement accompany the nurse for the shift Schedule visits during the daytime Conduct visits as quickly as possible Develop relationships with other residents in the neighborhood

Schedule visits during the daytime Visiting during the day is safer. It is not an effective use of law enforcement to have them follow the nurse throughout the shift. Visits should not be rushed but should be done efficiently. It is potentially unsafe, as well as impractical, to develop relationships with other residents unless they have an association with the client.

The nurse is attending an in-service about healthcare-acquired infections (HAIs). Which factor is considered a common cause of HAIs for clients in the acute care hospital setting? Inadequate fluid intake over 72 hours Decreased mobility for a week or longer Presence of an indwelling urinary catheter Undergoing a surgical procedure

Presence of an indwelling urinary catheter Rationale: Catheter-associated urinary tract infections (CAUTIs) are one of the more common healthcare-acquired infections (HAIs) in the acute care hospital setting. Surgical site infections, bloodstream infections, and pneumonia are other types of HAIs but are less common than CAUTIs.

The nurse is caring for a client with hepatitis C. Which infection precautions should the nurse implement? Airborne precautions Standard precautions Droplet precautions Transmission-based precautions

Standard precautions Rationale: Hepatitis C is transmitted via blood. Standard precautions are used for all blood-borne infections. Droplet precautions, transmission-based precautions and airborne precautions are not indicated for hepatitis C.

The nurse is teaching a client who uses a wheelchair for mobility about reducing pressure injuries. Which exercise should the nurse include in the teaching? Wheelchair push-ups Leg lifts Ankle rolls Shoulder lifts

Wheelchair push-ups Rationale: When sitting in a wheelchair, the majority of the bodyweight is on the pelvis and the buttocks. These are bony areas that can have skin breakdown if the pressure is not relieved. Wheelchair push-ups take pressure off the bony areas. The other exercises will maintain muscle tone but will not take pressure off the buttocks and pelvis.

The nurse is caring for a client who has just had an internal radiation therapy implant placed. Which of the following actions by the nurse is appropriate to ensure safety? Educating visitors to sit on the other side of the room Allowing the client's support person to stay overnight Assigning the client to a shared hospital room Ensuring that staff members wear a dosimeter badge

Ensuring that staff members wear a dosimeter badge Rationale: For the client undergoing internal radiation therapy, the nurse and all other care staff should wear a dosimeter badge to monitor the amount or radiation that they are exposed to. Typically, visitors are asked to be in the room for 30 minutes or less, therefore overnight stays are not considered safe. Visitors maintain a distance of 6 feet or more from the client. This client should be assigned to a private room.

The nurse is educating a group of parents about accidental poisoning of children. Which type of accidental poisoning is common in children under the age of six years? Topical contact Inhalation Eye splashes Oral ingestion

Oral ingestion Rationale: The greatest risk for young children is from oral ingestion. While children under age six may come in contact with other poisons or inhale toxic fumes, these are not as common.

The school nurse is teaching a group of teenagers about the prevention of sexually transmitted infections (STIs). Which statement by one of the students indicates an understanding of the teaching? "There are vaccines available that will prevent the majority of STIs." "Having multiple sexual partners puts me at a higher risk for an STI." "Being on birth control will prevent getting an STI." "Wearing a condom will eliminate any risk of contracting an STI."

"Having multiple sexual partners puts me at a higher risk for an STI." Rationale: While educating individuals on sexually transmitted infections (STIs) and prevention, discussing the risk of exposure should be emphasized. Although the use of condoms has been shown to reduce the risk of infection for both men and women, they do not completely eliminate the risk. The nurse should follow up on the other responses, as they indicate a lack of understanding of how an STI is transmitted or prevented.

The nurse is teaching an older adult client about measures to reduce incidence of falls in the home. Which of the following statements by the client indicates a need for further teaching? "I will plan on having grab bars installed in my shower." "I should plan on removing my wall-to-wall carpeting in my home." "I will install nightlights in my hallways and bathrooms." "I should remove clutter from the floor in my living areas."

"I should plan on removing my wall-to-wall carpeting in my home." Rationale: The client requires further teaching if the client states that wall-to-wall carpeting needs to be removed. The nurse should teach the client that loose rugs are tripping hazards and should be removed. Wall-to-wall carpeting is not known to increase fall risks in clients. Clients should be taught to remove clutter from living areas, provide adequate lighting at night, and install grab bars in showers to reduce the risk of falling

An 8-year-old is admitted to the hospital for surgery. The child's parent reports the allergies listed below. Which of these allergies should alert the staff to implement system safeguards? Shellfish Mold Balloons Perfumed soap

Balloons Rationale: A reaction to balloons or rubber toys may indicate a latex allergy; children with a history of allergies to avocados, bananas, and kiwis may also be at risk for a latex allergy. Common hospital items that may contain latex include catheters and other tubing, stethoscopes, blood pressure cuffs, and even wheelchair tires. All personnel who have contact with the child must be aware of this allergy and use latex-free supplies and equipment. If the child is in a semiprivate room, latex precautions should be used for both clients.

The operating nurse is monitoring sterile procedures during a client's surgery. Which of the following observed by the nurse would require intervention? The surgeon holds the instruments above the waist. The instruments in the sterile field are at least two inches from the edge. Blood-soaked gauze is placed on the sterile field The sterile field is always in the view of the staff.

Blood-soaked gauze is placed on the sterile field The role of the nurse in the operating room is to monitor and maintain the sterile fields. The nurse will also evaluate that OR staff maintain sterility during the procedure. Contamination of the sterile field, such as from used equipment, visibly soiled supplies, or someone crossing the sterile field, requires the nurse to intervene. To maintain sterility, instruments and supplies should be kept above waist level. The sterile field should be in constant view of staff to prevent unwitnessed contamination. A sterile field has a border of two inches from the edge that is not sterile.

An oncology nurse arrives to the unit and notes the staffing assignment indicates "float to ICU." What action does the nurse take? Prepare to receive report Clarify the duties to be performed Refuse the assignment Contact the chief nursing officer

Clarify the duties to be performed Floating to another unit does not indicate that the nurse will be expected to perform outside of their scope of practice. The nurse should clarify what duties are expected before accepting the assignment. The nurse should not receive report and assume care of the clients before knowing what duties are expected. Refusal of an assignment should not occur until the nurse evaluates the expected duties are within the scope of practice. Contacting the chief nursing officer does not follow the chain of command for reporting staffing concerns.

The nurse is preparing to obtain vital signs for assigned clients and notes an available blood pressure machine right outside of a room on contact isolation for Clostridium difficile. What is the appropriate action? Find the client's nurse and ask if the machine has been in the room Take the machine and begin the assessments Clean the machine with bleach wipes before starting Use alcohol-based hand sanitizer before moving the machine

Clean the machine with bleach wipes before starting Rationale: C. difficile is a gram-positive, anaerobic, spore-forming bacterium that is a common cause of diarrhea. It is not killed by alcohol-based hand-rubs. Washing hands frequently is an important infection control intervention. Regardless, washing hands before touching dirty equipment would not prevent the nurse from transferring C. difficile to another surface. The appropriate action is to proactively disinfect the machine using a bleach-based disinfecting wipe. Finding the nurse is nonproductive, as they may not have left the machine there or may not remember if it was cleaned.

The nurse is caring for a client who is confused and has repeatedly attempted to pull out their intravenous lines and feeding tube. The nurse receives an order from the health care provider (HCP) to apply soft wrist restraints. Which actions by the nurse are appropriate? Select all that apply.

Conduct a thorough physical assessment of the client Document that alternative interventions were attempted Explain the rationale for the use of restraints to the client Tie the wrist restraints using quick-release knots Rationale: Restraints should only be used as a last resort. If necessary, the least restrictive device should be used to restrain a client. Situations that require the use of restraints include when clients interfere with treatment (e.g., enteral feedings, intravenous infusions, etc.). Every two hours, restraints must be removed to assess skin integrity, allow for range of motion, and assess neurovascular status. Even though the client may be confused, the nurse must still explain the reason for applying restraints. Wrist restraints should be tied to a stationary part of the bed with a quick-release knot. A new restraint order must be written by the HCP every 24 hours. Prior to applying restraints, the nurse must conduct a thorough assessment of the client and document the events leading to the use of the restraint. The nurse should also document which alternatives to restraints were tried and the client's response to those measures.

The charge nurse observes a new staff nurse who is changing a sterile dressing. After removing the dirty dressing, the nurse removes the gloves and dons a new pair of sterile gloves before setting up the sterile field. What is the most appropriate action for the charge nurse to take? Inform the staff nurse that sterile gloves are not needed to remove the old dressing Hand the nurse a new pair of sterile gloves Discuss the dressing change technique with the nurse at the bedside Inform the nurse of the need to wash her hands after removal of the dirty dressing and gloves

Inform the nurse of the need to wash her hands after removal of the dirty dressing and gloves Rationale: The nurse should wash her hands after removing the soiled dressing and before donning the sterile gloves to clean and dress the wound. Not doing so compromises client safety and should be brought to the immediate attention of the nurse. Non-sterile gloves are adequate to remove the old dressing. A new pair of sterile gloves is not needed at this time. Discussion of the nurse's techniques should occur outside of the client's room.

A child is admitted to the pediatric unit with a diagnosis of suspected meningococcal meningitis. Which admission orders should the nurse implement first? Initiate droplet precautions Administer cefotaxime (Claforan) IV 50 mg/kg/day divided every six hours Monitor neurologic status every hour Institute seizure precautions

Initiate droplet precautions Rationale: Meningococcal meningitis is a bacterial infection that can be communicated to others. The initial therapeutic management of acute bacterial meningitis includes droplet precautions, initiation of antimicrobial therapy, monitoring neurological status along with vital signs, instituting seizure precautions and, lastly, maintaining optimum hydration. The first action is to initiate any necessary precautions to protect themselves and others from the potential infection. Viral meningitis usually does not require protective measures of isolation and these clients often return home to recover.

A nurse is reviewing the medical record for a client with a urethral stricture. Which prescription should the nurse clarify with the healthcare provider? Monitor intake and output Schedule a pelvic ultrasound Obtain a urine sample Insert an indwelling catheter

Insert an indwelling catheter Rationale: A urethral stricture is a narrowing of the urethra that restricts the flow of urine from the bladder. Insertion of an indwelling catheter may cause further trauma to the urethra if the stricture has not been dilated. Strict intake and output is not an invasive procedure and cannot cause harm to the client. A pelvic ultrasound is an expected diagnostic procedure to visualize the volume of urine present in the bladder. A urinalysis is an expected prescription to assess for signs of infection or blood in the urine.

The nurse has attended a staff education conference about incident reporting. Which of the following statements by the nurse indicates a need for further teaching? "An incident report should be submitted when a client chooses to leave a facility against medical advice." "An incident report should be submitted if a client is given the wrong medication." "An incident report should be submitted when a client develops respiratory arrest after receiving a prescribed opioid analgesic." "An incident report should be submitted if the wrong client is transported to the radiology department for an x-ray."

"An incident report should be submitted when a client chooses to leave a facility against medical advice." Rationale: An incident report is an agency record of an accident or unusual occurrence in the healthcare setting, which can be used to help prevent future incidents or accidents. It is the correct understanding of incident reporting if the nurse states that a medication error, a major adverse event such as respiratory arrest due to medical treatments, and a near-miss on an incorrect procedure on a client require incident reports. These are all considered errors or adverse/preventable events and should be reported to identify strategies to reduce human error and adverse events in the future. It indicates a need for further teaching if an incident report is submitted for a client who chooses to leave against medical advice. Clients have the right to refuse medical treatments at any time. The healthcare provider should be notified and proper documentation in the medical record regarding refusal of treatment is needed, however, this is not considered an adverse event that requires incident reporting.

The nurse has attended a staff education conference about incident reporting. Which of the following statements by the nurse indicates a need for further teaching? "An incident report should be submitted when a client develops drowsiness after receiving a prescribed opioid analgesic." "A medication that is given to a client at the wrong dosage requires the submission of an incident report." "A procedure that is started on the wrong client requires the submission of an incident report." "An incident report should be submitted for a client who fell after receiving a sedative."

"An incident report should be submitted when a client develops drowsiness after receiving a prescribed opioid analgesic." Rationale: An incident report is an agency record of an accident or unusual occurrence in the healthcare setting, which can be used to help prevent future incidents or accidents. It is the correct understanding of incident reporting if the nurse states that a client fall, medication error, and incorrect procedure on a client require incident reports. These are all considered errors or adverse/preventable events and should be reported to identify strategies to reduce human error and adverse events in the future. Clients may experience side effects to certain medications, and this requires monitoring and possible treatment depending on the severity, however, this is not considered an actual accident that requires incident reporting.

The nurse has attended a staff education conference about incident reporting. Which of the following statements by the nurse indicates a need for further teaching? "A healthcare provider should submit an incident report if an injury to a client is witnessed, even if they were not involved in the incident." "An incident report should be submitted when a client refuses a prescribed medication." "An incident report should be submitted if the wrong client receives a chest x-ray." "A nurse should submit an incident report if they recognize, after the fact, that a previous nurse performed a medication error."

"An incident report should be submitted when a client refuses a prescribed medication." An incident report is an agency record of an accident or unusual occurrence in the healthcare setting, which can be used to help prevent future incidents or accidents. It is the correct understanding of incident reporting if the nurse states that all healthcare workers should submit an incident report if an injury is witnessed, regardless of their involvement in the incident. It also demonstrates a correct understanding of incident reporting if an incident report is submitted if a client receives a wrong diagnostic treatment. Nurses should also submit an incident report if it is determined that an error occurred on a previous shift, even if they were not the ones to make the error. It indicates a need for further teaching if an incident report is submitted for a client who refuses medications. Clients have the right to refuse medical treatments at any time and the healthcare provider should be notified, however, this is not considered an adverse event that requires reporting.

The nurse is providing teaching about system internal radiation to a client diagnosed with thyroid cancer who will be treated with radioactive iodine. Which of the following statements will the nurse include in the teaching? "You may return to work immediately." "Avoid airplane travel after your treatment." "It is safe to sleep in the same bed as your spouse." "Limit the amount of fluids you drink for several days."

"Avoid airplane travel after your treatment." I-131 (radioactive iodine) is used to treat thyroid cancer. Clients are asked to follow some radiation precautions after treatment in order to limit radiation exposure to others, especially pregnant women and children. Radiation detection devices used at airports or in federal buildings may pick up even very small radiation levels. Therefore, travel should be avoided if possible. Clients should sleep in a separate bed (6 feet of separation) for 1-11 days, depending on dosing. Clients cannot return to work immediately, as they need to remain socially distanced. Fluids are encouraged in the first 2-3 days after treatment.

The nurse has attended a staff education conference about internal radiation therapy safety. Which of the following statements by the nurse indicates a need for further teaching? "Caregivers should maintain a 3-foot distance from clients with radiotherapy implants unless providing direct client care." "Visitors should be limited to 30-minute visits each day to reduce time exposed to radiation." "Portable lead shields are effective in reducing caregiver exposure to radiation during client care." "Caregivers and visitors who may be pregnant should not enter the room of clients undergoing internal radiation therapy.

"Caregivers should maintain a 3-foot distance from clients with radiotherapy implants unless providing direct client care." Caregivers and visitors are at risk of excessive radiation exposure when caring for clients who are undergoing internal radiation therapy for cancer treatments because these patients are emitting radiation during treatment. Proper safety precautions to reduce exposure include limiting time, increasing distance from the client, and implementing shielding. The correct distance that should be maintained is at least 6 feet from the client to reduce exposure to radiation. Visitors and staff should limit their time with the client to 30 minutes, pregnant staff and visitors should not enter the room due to potential risk to the fetus, and lead shields should be implemented to provide a barrier to radiation exposure.

The nurse has attended a staff training program about client triage during a disaster. Which of the following statements by the nurse indicates a correct understanding of the training? "Clients with injuries such as airway obstruction or shock should be classified as emergent and seen immediately." "Clients with injuries such as open fractures or large wounds should be classified as non-urgent and can generally be seen in more than 2 hours." "Clients experiencing abrasions and contusions should be classified as expectant and may not be treated." "Clients with massive head trauma, extensive burns, or high cervical spinal cord injury should be classified as urgent and should be seen within 30 minutes to 2 hours."

"Clients with injuries such as airway obstruction or shock should be classified as emergent and seen immediately." Rationale: In mass casualty disaster situations, triage focuses on doing the greatest good for the greatest number of people. Therefore, when resources are severely limited, some clients who have very extensive critical injuries who would otherwise receive massive resuscitation efforts (cervical cord injury, head injuries, massive burns), may be classified as expectant, or black-tagged and allowed to die or not be treated. Clients with airway compromise or shock are classified as emergent and seen immediately. Clients with open fractures or wounds should be classified as urgent and seen within 30 minutes to 2 hours. Clients with abrasions or contusions should be classified as nonurgent and can be seen after 2 hours or when other more urgent clients have been attended to.

The nurse has attended a staff training program about nursing roles during disasters. Which of the following statements by the nurse indicates the need for further training? "Nurses may be responsible for activating notification systems to call in nurses who are not scheduled to work." "General staff nurses should be assigned to perform triage for incoming clients to determine level of severity of injuries." "Critical care nurses should be prepared to determine clients who are stable enough for transfer to ensure adequate beds for critically ill clients." "Administrative nurses may have roles reassigned to provide care for stable clients on the unit."

"General staff nurses should be assigned to perform triage for incoming clients to determine level of severity of injuries." Rationale: Mass casualty events such as disasters require nurses to assume roles outside of normal daily operations. Typically, general staff nurses will be assigned to discharge stable clients or care for stable clients in the emergency department to allow emergency department nurses to perform triage and critical care duties as needed for disaster victims. Nurses should be prepared to activate notification systems to increase nursing staff available to care for victims. Critical care nurses should discharge stable clients to medical-surgical units to allow for space for victims who are unstable. Administrative nurses may be reassigned outside of their regular roles to care for stable clients with predictable outcomes.

The nurse has attended a staff training program about recognition of potential bioterrorism in the community. Which of the following statements by the nurse indicates a need for further training? "An unusual geographic clustering of illness may indicate potential bioterrorism." "High incidence of an illness in an expected population may indicate bioterrorism." "An increase in emergency department clients who report similar symptoms may indicate bioterrorism." "The local health department should be notified of any unusual patterns identified in a facility that may indicate bioterrorism."

"High incidence of an illness in an expected population may indicate bioterrorism." Rationale: Health care workers need to have a heightened awareness of patterns that may indicate potential bioterrorism, such as unusual clustering of illness in a geographic area or an increase in emergency department visits of clients with similar symptoms. It requires further teaching if the nurse states that a high incidence of illness in an expected population indicates bioterrorism. A high incidence of illness in an unusual or unexpected population, such as chickenpox-like symptoms in adults instead of children, could indicate bioterrorism and such patterns should be reported to the local health department to determine if a larger pattern is occurring within the community.

A pre-school nurse is providing education to parents on poisoning prevention. Which statement made by a parent indicates the need for further teaching? "I have my furnace professionally inspected each year." "I keep the poison control center phone number on the counter." "I always store my cleaning products underneath the sink." "I dispose of medications in the trash mixed with coffee grounds."

"I always store my cleaning products underneath the sink." Rationale: Poisoning prevention in children includes keeping toxic chemicals out of reach. Storing cleaning products underneath the sink is not safe unless a cabinet has a safety latch. Furnaces should be professionally inspected each year to ensure good ventilation and proper escape of combustion byproducts. The poison control center phone number should be kept within reach at all times in case of accidental ingestion. Disposing medications mixed with coffee grounds or cat litter discourages children from ingesting substances.

A nurse has attended a conference about communicable diseases. Which statement by the nurse indicates the need for further teaching? "I only need to report diseases associated with children." "I should review the infection control policies." "I need to report suspected communicable disease to the infection control department." "I need to use the correct isolation precautions for the disease."

"I only need to report diseases associated with children." Rationale: The nurse should be aware of the infection control policies for their organization. The CDC recommends that all suspected communicable diseases be reported to the health department via the infectious disease department. Staff members need to use the appropriate infection control precautions for all communicable diseases. All communicable diseases need to be reported, not just those associated with children.

The nurse on a psychiatric unit is speaking with an unlicensed assistive person (UAP) who was recently hired. The UAP has been assigned to monitor an older adult client who has been making inappropriate sexual comments about staff and other clients. Which statement by the UAP indicates that further clarification from the nurse is needed? "I think the client should be placed in the seclusion room." "I will assist the client with using the bathroom as needed." "The client is requesting medication to help them sleep." "I have been reading to the client which seems to help."

"I think the client should be placed in the seclusion room." Rationale: None of the behaviors described indicate that the client is a physical threat to themselves or others that may require the client to be restrained or placed in seclusion. Furthermore, it is outside of the UAP's scope and responsibilities to suggest that the client should be placed in seclusion. Therefore, the UAP's statement about placing the client in the seclusion room should be clarified by the nurse.

The nurse reviews the home care instructions with a parent of a 3-year-old client who has pertussis. Which statement by the parent indicates a need for further teaching? "I know that my child will make a loud whooping sound." "I understand this whooping cough is viral, and I have to let it run its course." "I understand that I need to watch for respiratory distress signs with pertussis." "Coughing spells can be triggered by dust or smoke."

"I understand this whooping cough is viral, and I have to let it run its course." Rationale: Pertussis is caused by the bacteria Bordetella pertussis and treatment requires antimicrobial therapy. Pertussis is transmitted by direct contact or respiratory droplets from coughing. The communicable period occurs primarily during the catarrhal stage. Symptoms of pertussis consist of a respiratory infection followed by increased severity of cough with a loud whooping on inspiration. The child may experience respiratory distress, and the parents should be instructed on reducing environmental factors that cause coughing spasms, such as dust, smoke, and sudden changes in temperature.

A nurse is providing education to a client on the safe use of a breast pump. Which client statement indicates the need for further teaching? "I will inspect the cord of my electric breast pump for any exposed wires." "I will turn off the breast pump before removing my breast shield." "I will gradually increase the speed setting on my breast pump." "I will clean the breast pump at the end of every day."

"I will clean the breast pump at the end of every day." Breast pumps should be sanitized after every use to avoid the risk of infection. Electric breast pumps require a power source and should be inspected with every use. Exposed wires increase the risk of electric shock. The breast pump should be turned off before attempting to break the seal between the breast and the shield to avoid tissue injury. A gradual increase in the speed of the breast pump ensures comfort and decreases the risk of tissue injury.

The nurse is teaching the parents of a toddler-age client about home safety. Which of the following statements by the parent indicates the need for further teaching? "I will place hot cooking pans on the back burner of the stove with handles pointed inward." "I will ensure my houseplants are kept very low to the ground or on the floor." "I will keep all objects with sharp edges, such as knives, out of my child's reach." "I will keep all cleaning solutions and medicines in a locked cabinet."

"I will ensure my houseplants are kept very low to the ground or on the floor." The nurse should provide additional teaching if the parent states that houseplants should be placed on the floor. Toddlers are curious and often stick objects into their mouths, including plants, which could cause unintentional poisoning. Houseplants should be kept out of reach and off surfaces that could cause them to get knocked over. It indicates a correct understanding of the teaching if the client keeps pots on the back burner of the stove, keeps sharp objects out of the client's reach, and keeps all cleaning solutions in a locked cabinet.

The nurse is teaching the parents of a school-age client about home safety. Which of the following statements by the parent indicates the need for further teaching? "I will ensure my child wears reflective clothing when walking at night." "I will ensure my loaded firearm is kept in a locked cabinet." "I will ensure my child wears a helmet while using a bicycle." "I will ensure my child is supervised when using electrical appliances and tools."

"I will ensure my loaded firearm is kept in a locked cabinet." Rationale: It requires additional teaching if the parent states that a loaded firearm is kept in a locked cabinet. While it is correct to lock firearms, they should be kept unloaded and the ammunition should be kept in a separate location from the firearm. It indicates a correct understanding of the teaching if the parent ensures their child wears reflective clothing at night, wears a helmet while bicycling, and is supervised while using electronic tools.

A nurse is providing discharge instructions on safety precautions to a client newly diagnosed with epilepsy. Which client statement indicates the need for further teaching? "I will make sure someone is always with me when I drive." "I will pad the hard edges of furniture." "I will wear a medical identification tag at all times." "I will take showers instead of baths."

"I will make sure someone is always with me when I drive." Rationale: Driving laws differ in every state, with some of them restricting or limiting driving for people who have a history of recent seizure activity. The nurse should encourage the client to research state driving laws for safety purposes. Padding hard edges of furniture decreases the risk of injury should a client have a seizure that results in a fall. A medical identification tag is encouraged. Medical identification tags can alert bystanders of a medical condition in case of an emergency. Taking a bath increases the risk of a submersion injury should a seizure with loss of consciousness occur.

The nurse is providing education at a prenatal class about car seat safety. Which client statement indicates understanding of the use of the car seat? "I can use the car seat that my mom used for me when I was a child." "My infant can ride in a rear-facing seat in the front seat of my car." "I will make sure that the car seat is secured with the safety belt." "I will make sure my car seat is fastened upright."

"I will make sure that the car seat is secured with the safety belt." Rationale: When installing a car seat, follow the manufacturer's instructions on how to secure the seat with the car's seat belt. After 10 years, car seats are considered expired. Car seats should not be placed in the front seat due to the risk of airbag deployment. Infant seats need to be in a semi-reclining position to prevent their chin from falling on their chest to maintain an open airway.

The charge nurse working at the nurse's station overhears a conversation between a new graduate nurse and an unfamiliar employee who is wearing appropriate identification. The employee is asking for the security code to enter the medication room. Which of the following statements should the charge nurse make? "Only nursing staff is permitted in the medication room." "I will need to know the reason for your access to the medication room." "I will enter the code to the medication room for you." "Someone will need to be in the medication room with you."

"I will need to know the reason for your access to the medication room." Rationale: Organizations must ensure the medications are secure - meaning protected from unauthorized access, tampering, theft, or diversion. Medication rooms are accessed by authorized personnel, which can include pharmacists, pharmacy technicians, nurses, and other licensed personnel. Therefore, it is inappropriate to teach the nurse that only nursing staff members are authorized to be in the room. However, the individual needs to be vetted to determine if they are included in the individuals with approved access. It is not an effective use of time to supervise an authorized individual as they complete their work. While the employee may need regular access to the room, it may not be appropriate to provide the code until further information can be obtained.

The nurse is educating a client who has diabetes mellitus on home safety. Which of the following statements by the nurse is appropriate? "Store used needles for later use." "Keep a spare vial of insulin in the refrigerator." "You don't need to check your glucose if your diabetes is well-controlled." "Don't wear shoes while inside your home."

"Keep a spare vial of insulin in the refrigerator." Rationale: Having a backup vial of insulin in the refrigerator is a safe practice if the current vial becomes lost or damaged. Used needles should be discarded in a sharp-safe container. Glucose levels are important to assess regardless of how well the client's condition is controlled. The client should wear well-fitting shoes as often as possible, even indoors, to prevent foot injury.

A nurse is providing education to a client on the safe use of oxygen equipment at home. What will the nurse include in the teaching? "Transport the oxygen tank on its side when travelling by car." "Clean the nasal cannula tubing with an alcohol-based product." "Use oxygen tubing that is less than 50 feet in length." "Keep the oxygen concentrator 6 feet away when you are cooking."

"Keep the oxygen concentrator 6 feet away when you are cooking." Rationale: Oxygen equipment should be kept at least 6 feet away from any source of fire to decrease the risk of combustion. Oxygen tanks should always be kept upright to decrease the risk of oxygen escaping and causing combustion. Alcohol is flammable and can cause a fire. Tubing can be up to 30 meters (98.4 feet) in length to allow the client freedom of movement in the home.

The nurse has attended a staff training program about nursing roles during disasters. Which of the following statements by the nurse indicates a correct understanding of the training? "Nurses should be prepared to work on different units during a disaster." "General staff nurses should be assigned to perform triage for incoming clients to determine level of severity of injuries." "Nurses who are not scheduled to work should remain at home during a disaster." "A nurse should expect to perform duties outside the nursing scope of practice during a disaster."

"Nurses should be prepared to work on different units during a disaster." Rationale: Nurses should be prepared to perform roles outside of their regular duties during a disaster, however this does not include acting out of scope for nursing practice. General staff nurses should be assigned to care for stable clients in the emergency department, thus allowing emergency department nurses to perform critical triage duties. Nurses who are not scheduled to work should be ready to report to their facility to assist with any duties that may be needed.

The nurse has attended a staff education conference about electrical safety in the healthcare setting. Which of the following statements by the nurse indicates the need for further teaching? "Plugs should be removed from the wall outlet by pulling from the cord." "All electrical equipment should be checked for fraying or other signs of damage before use." "Electrical equipment should not be used near wet areas such as sinks and showers." "Electrical cords should be kept coiled or taped to the ground to prevent damage and tripping hazards."

"Plugs should be removed from the wall outlet by pulling from the cord." Rationale: It indicates a correct understanding of electrical safety if the nurse states that cords should be kept coiled or taped to the ground to prevent tripping and damage to the cords, to avoid the use of electrical equipment around wet areas since water is an electrical conductor, and to check cords for fraying and signs of damage, which could increase the risk of electrical shock to the user. Plugs should be removed from the wall outlet by pulling from the plug, not the cord, as this could cause damage to both the outlet and the device, increasing the risk of electrical shock.

A nurse is providing education to a client post-knee surgery on safe use of a continuous passive motion (CPM) machine. What will the nurse include in the teaching? "Position the knee where the CPM machine bends." "Check the range of motion settings every other day." "Set the degree of flexion where it feels the most comfortable." "Remove any padding between your knee and the CPM machine."

"Position the knee where the CPM machine bends." Rationale: The client's leg should be positioned so the affected joint is in line with the bend of the continuous passive motion (CPM) machine. This promotes proper flexion of the joint. The cycle and range of motion settings should be checked at least every 8 hours to ensure the degree of flexion is as prescribed. The degree of flexion should be set according to the provider's prescription. The CPM machine should be well padded to avoid skin integrity issues.

The nurse has attended a staff training program about recognition of personal risks while responding to external disasters. Which of the following statements by the nurse indicates a need for further training? "Disaster sites should be assessed for the potential to cause injury due to structural collapse." "Responders to a terrorist attack are at a reduced risk of becoming a target of a secondary attack." "Exposure to contagious diseases and natural hazards may occur while responding to disasters." "Responders should always be aware of the risk of interpersonal violence in unsecured areas."

"Responders to a terrorist attack are at a reduced risk of becoming a target of a secondary attack." Rationale: Responders to external and internal disasters must be aware of the personal risks involved. It requires additional training if the nurse states that responders are at reduced risk of becoming the target of a secondary terrorist attack. Responders to potential terrorist attacks must always be vigilant of a second attack, which is aimed at harming first responders. Disaster sites should be assessed for potential structural collapse, which can cause injury to responders. Responders should take appropriate personal protective equipment if possible, based on contagious or natural hazards that may be encountered, and should be vigilant about personal safety and security regarding violence in unsecured areas.

The emergency room nurse is caring for a child suspected of poisoning. The child's parent asks the nurse what the purpose of the prescribed activated charcoal is. How should the nurse respond? "This substance removes the poison from the body through the urinary system." "This substance inactivates the toxins that your child ingested." "The activated charcoal binds with the poison to limit absorption from the digestive tract." "The activated charcoal induces vomiting to remove the poison.

"The activated charcoal binds with the poison to limit absorption from the digestive tract." Rationale: Activated charcoal may be used for accidental poisoning, as it keeps swallowed substances from being absorbed from the gastrointestinal tract into the bloodstream. Its action is to bind to other substances on its surface (adsorption), pushing the poison through the digestive system faster and decreasing the amount absorbed into the body. It does not inactivate the poison but rather binds to it in the digestion system to be removed more quickly.

The parents of a 3-year-old toddler ask the nurse how long their child will have to sit in a child car seat while in an automobile. Which is the best response by the nurse? "The child must be 5-years-old to use a regular seat belt." "The child must reach a height of 50 inches (127 cm) to use a regular seat belt." "The child can use a regular seat belt when they can sit still." "The child should use a car seat for as long as possible."

"The child should use a car seat for as long as possible." Rationale: The American of Academy of Pediatrics (AAP) recommends that all children should ride in a rear- or front-facing seat for as long as possible or until they reach the highest weight or height allowed by the car seat manufacturer. Toddlers who have outgrown the weight or height limit for the rear-facing car seat should use a forward-facing car seat. Many seats can accommodate children up to 65 pounds (29 kg) or more.

The nurse is preparing a client for a prescribed MRI. The client states "If I get anxious during the procedure, how will I notify staff?" Which statement would be appropriate for the nurse to make? "There will be a microphone in the machine for you speak to the staff." "Just raise your hands to get the attention of the staff." "Press the button to move out of the machine." "A staff member will be in the room with you."

"There will be a microphone in the machine for you speak to the staff." Rationale: MRI uses a magnet to get images. The machine is loud. MRI machines have a two-way microphone to communicate with the staff. The client needs to remain still so the images are not distorted. Staff members do not remain in the room during the procedure.

The nurse is providing education to a client who works on an assembly line about preventing repetitive stress injuries. Which of the following statements should be included in the teaching? "Use a footrest at the workstation." "Use the lunch period to elevate the legs." "Twist at the waist when possible." "Limit the amount of walking done during the work shift."

"Use a footrest at the workstation." Rationale: Prolonged standing can lead to lower back repetitive stress injuries. Using a footrest reduces intravertebral disc stress by preventing excessive lordosis. Frequent work breaks should be used to reduce back stress instead of a single, prolonged break. Workers should pivot instead of twisting at the waist. Walking is encouraged to promote venous return and circulation and reduce tension.

The nurse is completing a home health visit for a client who experiences right sided-weakness due to a previous cerebrovascular accident. Which of the following statements by the nurse is an appropriate safety recommendation? "Use a long-handled bath brush while showering." "Using a bedside commode will cause further loss of mobility." "Avoid using your weak side for self-care practices." "Only complete range of motion activities with assistance."

"Use a long-handled bath brush while showering." Rationale: The client who experiences hemiplegia will benefit from modified self-care devices, such as a long-handled shower brush. These devices reduce the risk of falls and injuries. Using a bedside commode may be necessary and will not cause mobility loss. The client should use the affected side as much as possible. The client should complete range of motion activities with the caregiver but can do these activities independently as well.

The nurse is providing safety and accident prevention instructions to the parents of 4-year-old twins. Which statement by the parents indicates a correct understanding of the instructions? "Accidents can happen. We will make sure to call 911 right away." "We are having a fence installed around the backyard pool." "It is normal for little children to be curious and want to explore." "The children are old enough to sit in the car without a child safety seat."

"We are having a fence installed around the backyard pool." Rationale: In the United States (US), accidents are the leading cause of death in small children. Common accidents include choking and suffocation, falls, drowning, and poisoning. Nurses must teach parents how to promote a safe environment for their children. When there is a pool at the home, it is recommended, and can be required by law, to have a fence and lockable gate installed around the pool to prevent unsupervised access and accidental drowning. The other statements do not indicate a correct understanding of accident prevention for children.

A home care nurse is evaluating the home situation for a client with Alzheimer's disease. Which statement below, by a family member, should be reinforced by the nurse? "We go to a group discussion every week at our community center." "At least two full meals a day should be eaten." "We have safety bars installed in the bathroom and have alarms on the exterior doors." "To take the medication three times a day is not a problem."

"We have safety bars installed in the bathroom and have alarms on the exterior doors." Rationale: For clients with Alzheimer's disease, safety is the primary concern. The nurse should reinforce the use of safety bars in the bathroom and the alarms on the exterior doors. The other actions are also positive actions for the family in the care of a family member with Alzheimer's disease.

The nurse is educating a group of individuals about how to prevent hepatitis B and C. Which statement by the nurse would best describe the prevention of these two diseases? "You can receive a yearly vaccination to prevent the diseases." "You should use protection when engaging in sexual intercourse." "You should talk to your health care provider when traveling internationally." "You can eat fresh fruit picked from the tree without the need to wash."

"You should use protection when engaging in sexual intercourse." Rationale: Hepatitis B and C are considered bloodborne illnesses that can be spread by contact with infectious bodily fluids and blood. Preventative measures should include using safer sex practices, avoiding risky behaviors, and sharing needles. There is not a yearly vaccine for these diseases. Individuals in the U.S. receive the hepatitis B vaccination by 6 months of age. There is no vaccine for hepatitis C. When traveling internationally to locations where sanitation may be a concern, the individual might be at risk for contracting hepatitis A, not hepatitis B or C.

The nurse is conducting a community-wide seminar on childhood safety issues. Which child is at the highest risk for poisoning? A 15-year-old who likes to repair bicycles A 10-year-old who occasionally stays at home unattended A 20-month-old who has just learned to climb stairs A 9-month-old who stays with a babysitter five days a week

A 20-month-old who has just learned to climb stairs Rationale: Poisoning is one of the most common health care emergencies encountered with young children. Toddlers, aged 1-3 years (e.g., 20-months-old), are at the highest risk for poisoning. Toddlers are increasingly active, curious, and anxious to explore. They are too young to know what is dangerous. Additional risk factors for poisoning include improper storage of toxins, children spending more time in unfamiliar settings and caregiver distraction.

A nurse is asked to recommend one of her assigned clients for discharge due to a disaster plan activation. Which client will the nurse recommend for discharge A client with a platelet count of 70,000/mm³ and history of systemic lupus erythematosus A client with a potassium level of 6.2 mEq/L and history of kidney disease A client with a PaO₂ of 55 mmHg and history of chronic obstructive pulmonary disease A client with a hemoglobin level of 10 g/dL and history of chronic anemia

A client with a hemoglobin level of 10 g/dL and history of chronic anemia Rationale: A client with a hemoglobin of 10 g/dL is the most stable client for discharge. The normal hemoglobin level is 12 to 18 g/dL. A hemoglobin level of 10 g/dL is not uncommon with chronic anemia. A client with a low platelet count is not appropriate for discharge. Thrombocytopenia can lead to uncontrolled bleeding. A potassium level of 6.2 mEq/L is not a safe discharge. The normal potassium level is 3.5 to 5.0 mEq/L. Hyperkalemia can cause cardiac arrhythmias. A client with a partial pressure of oxygen (PaO₂) level of 55 mmHg is not a stable client for discharge. Hypoxemia can lead to decreased tissue perfusion.

The hospital has sounded the call for a disaster drill on the evening shift. Which client would be most appropriate to be discharged to make room for a new admission? A client with type 2 diabetes who was admitted for acute cellulitis of the lower leg 48 hours ago A client newly diagnosed with type 1 diabetes who was admitted with antibiotic-induced diarrhea 24 hours ago A client with a history of lupus who was admitted with Stevens-Johnson syndrome that morning A client with a history of being ventilator-dependent who was admitted with pneumonia eight days ago

A client with a history of being ventilator-dependent who was admitted with pneumonia eight days ago Rationale: The best candidate for discharge is the one who has a chronic condition and has an established plan of care. The client who has been on the ventilator for some time is most stable and could continue medication therapy at home or their residence; therefore, this client is the most appropriate client to be discharged. The other clients are too acutely ill and have a risk for instability. A client newly diagnosed with diabetes who has an active infection is at risk for developing diabetic ketoacidosis. This client is also at risk for fluid loss with dehydration and electrolyte imbalance. A client newly diagnosed with acute cellulitis may require intravenous (IV) antibiotics. Stevens-Johnson syndrome (SJS) is a hypersensitivity reaction that involves the skin and mucous membranes. Clients with SJS are treated similarly to burn victims, as they are at high risk for fluid loss and are susceptible to infections.

During a disaster activation plan, a nurse is tasked with recommending the most stable client for discharge. Which of the assigned clients will the nurse recommend? A client with tuberculosis who is on airborne precautions A client with a perirectal abscess who requires daily wound care A client with an external fixator to the left lower extremity who is homeless A client with pancreatic cancer who is post-operative Whipple procedure

A client with a perirectal abscess who requires daily wound care A client with a perirectal abscess who requires daily wound care is the most stable client. Wound care can be taught to the client's family or caregiver prior to discharge. A client with tuberculosis on airborne precautions is not appropriate for discharge. Community spread is possible if the client is discharged without proper treatment. Clients with invasive devices such as a fixator are at a high risk for infection. A client who is homeless requires a consult from case management and social work. A client who had a Whipple procedure is not stable for discharge. A Whipple procedure is a complex surgery that requires extensive medical management.

A charge nurse on a stroke unit receives a request for several client admissions. Which client will the charge nurse assign to a room in front of the nurses' station? A client with dysphagia A client with hemianopsia A client with expressive aphasia A client with agnosia

A client with hemianopsia Hemianopsia is vision loss in one or both eyes. Decreased sensory perception increases the client's risk for falls. Expressive aphasia is difficulty with speaking or finding the correct words to verbally communicate. Clients with expressive aphasia benefit from picture-based communication boards. Dysphagia is difficulty swallowing. Precautions related to dysphagia include elevating the head of the bed and assisting the client with meals. Agnosia is the inability to recognize familiar objects, people, or places. Clients with agnosia are still able to comprehend safety instructions.

A community health nurse is educating a group of clients on burn safety. Which client is at a higher risk for burns? A client with peripheral arterial disease A client with chronic kidney disease A client with a traumatic brain injury A client with diabetic nephropathy

A client with peripheral arterial disease Rationale: Clients with peripheral arterial disease have decreased blood flow and sensitivity to the lower extremities. Clients with decreased sensation to the feet can suffer burns without realizing the injury. Decreased sensation is not an expected finding for a client with chronic kidney disease. Clients with a traumatic brain injury are at a high risk for falls and seizures. Diabetic nephropathy is damage to the kidneys due to elevated blood glucose levels. Diabetic nephropathy does not result in decreased sensory perception.

The nurse is documenting an occurrence in which a surgical tool was left in a client's abdomen following a procedure. Which type of practice error should the nurse document as having occurred? A never-event A sentinel event A near-miss event An unpreventable event

A never-event Rationale: The nurse should document the retention of a surgical object in a client following a procedure as a never-event. Never-events are defined by the Joint Commission as surgeries on the wrong body part, foreign objects placed in the client after surgery, and mismatched blood transfusions. A sentinel event is one in which serious injury or death occurred due to errors. A near-miss event is the identification of any event or situation that might have resulted in client harm, but the harm did not occur due to timely intervention by healthcare staff. An unpreventable event is one in which death or client injury would occur in the absence of a medical error.

The infection control nurse is evaluating the infection prevention procedures on the unit. Which of the following observed by the nurse would require intervention? The nurse puts on a mask, a gown, and gloves before entering the room of a client on strict isolation. A client with active tuberculosis is asked to wear a mask when he leaves his room to go to another department for testing. A nurse with open, weeping lesions on the hands puts on gloves before giving direct client care. Staff are not wearing gloves when feeding clients in the common dining area.

A nurse with open, weeping lesions on the hands puts on gloves before giving direct client care. Rationale: Persons with exudative lesions or weeping dermatitis should not give direct client care or handle client-care equipment until the condition resolves; this helps prevent the spread of any pathogens. There is no need to wear gloves when feeding a client. However, universal precautions (treating all blood and body fluids as if they are infectious) should be observed in all situations. A client with active tuberculosis should be on respiratory precautions. Having the client wear a mask when leaving his private room is appropriate to prevent exposure to others. Strict isolation requires the use of a mask, a gown, and gloves.

A charge nurse is assigning new admissions to the surgical unit. Which client will the charge nurse assign to the room across from the nurses' station? A postoperative client with history of schizophrenia A client transferred from the ICU with severe pain A postoperative client with symptoms of delirium A client transferred from the ED with a chest tube

A postoperative client with symptoms of delirium Rationale: Postoperative delirium can occur as a result of general anesthesia. The client may experience agitation and hallucinations. Clients with delirium require frequent visual checks to ensure safety. A client with a history of schizophrenia does not require special accommodations unless actively experiencing symptoms of the illness. A client with severe pain does not need to be assigned close to the nurses' station. The client should be provided instructions on how to call for assistance. A client with a chest tube does not require frequent visual checks. The nurse should assess the client and the chest tube as per protocol.

The nurse is providing staff education on security measures in the newborn nursery. All the babies in the unit have electronic tags attached to their umbilical cord clamps. What information about these tags will the nurse include in the training? A sensor activates an alarm if the newborn is taken out of the unit. The newborn's identification can be quickly identified using the tag. Medication administration is performed after the tag's barcode is scanned. The parent's identification band is matched to the umbilical tag.

A sensor activates an alarm if the newborn is taken out of the unit. Prevention of infant abductions has been successful through a combination of increased security measures in hospitals, including video cameras and alarm devices, and education of staff and parents about precautions to take while in the hospital. Umbilical tags have electronic sensors that will activate audible alarms if the baby is taken outside of the unit. Newborn identification is done using an ID bracelet. Medication administration may utilize barcode scanning, but this is not done using the umbilical tag. The parent's identification band is matched to the arm or ankle ID bracelet.

The nurse is documenting an occurrence in which a client fell during a transfer and resulted in an intracerebral hemorrhage and death. Which type of practice error should the nurse document as having occurred? A sentinel event A near-miss event A never-event An unpreventable event

A sentinel event Rationale: The nurse should document the death of a client following a fall during a transfer on the unit as a sentinel event. A sentinel event is one in which serious injury or death occurred due to errors. Never-events are defined by the Joint Commission as surgeries on the wrong body part, foreign objects placed in the client after surgery, or mismatched blood transfusions. A near-miss event is the identification of any event or situation that might have resulted in client harm, but the harm did not occur due to timely intervention by healthcare staff. An unpreventable event is one in which death or client injury would occur in the absence of a medical error.

A nurse is assessing environmental safety in the home of a client with a toddler. Which observation by the nurse would require immediate follow up? A pile of paper bags in the living room A swimming pool in the backyard A jewelry box on a high-top counter A stairwell cluttered with shoes

A swimming pool in the backyard Rationale: Unintentional drowning is one of the leading causes of suffocation in children. A swimming pool is a high-risk area for toddlers. Identifying environmental risks can prompt the nurse to educate on water safety. Plastic bags as opposed to paper are a high risk for suffocation. Small items, such as jewelry or toys, should be kept out of the toddler's reach. A high-top counter is an acceptable area. A cluttered stairwell is a high risk for falls, not suffocation.

The nurse working in pediatrics is admitting a client with a diagnosis of measles. Which of the following transmission-based precautions should be instituted? Airborne Contact Droplet Reverse

Airborne Rationale: The client with measles should be placed on airborne precautions. Patients with measles should remain in Airborne Precautions for 4 days after the onset of rash (with the onset of rash considered to be Day 0). Standard precautions should be adhered to, as is the case for all patients. Reverse isolation is designed for immunosuppressed clients.

The nurse is planning care for a pediatric client with rubeola. Which type of precautions should the nurse implement for this client? Contact Droplet Airborne Neutropenic

Airborne Rubeola is transmitted by air with infectious aerosolized particles, which are micro-droplets that suspend in the air and travel greater distances. Airborne precautions would require the use of an N95 mask in addition to the gown and gloves. Droplet precautions are implemented for clients with respiratory infections that suspend in larger droplets, which do not suspend in the air and travel less than 3 feet. Contact precautions would require direct contact with the organism such as stool with E. coli. Bloodborne precautions are for diseases transmitted by blood, such as hepatitis. Neutropenic precautions are for clients with low WBCs.

The nurse in an assisted living facility is reviewing the medical record of an older adult client who has had several falls. Which nursing problem is the priority for this client? Inadequate nutritional intake related to anorexia Ineffective coping due to loss of independence Hopelessness related to death of the spouse Alteration in sensory perception due to impaired vision

Alteration in sensory perception due to impaired vision Rationale: Safety is crucial in the health maintenance of older adults. When compared to younger adults, older adults are at higher risk for accidents and falls due to normal sensory changes, slowed reaction time, decreased thermal and pain sensitivity, changes in gait and balance, and medication effects. Most accidents such as falls occur in or around the home. Vision impairment caused by age-related changes will lead to impaired sensory perception that increases the person's risk for falls. Therefore, alteration in sensory perception due to impaired vision is the priority nursing problem.

The homecare nurse is caring for a client on home oxygen. Which of the following findings requires an immediate further assessment? An ashtray is present on the side table. The heating system in the home is natural gas forced air. The oxygen concentrator is plugged into a surge protector. Replacement air filters are present on the kitchen counter.

An ashtray is present on the side table. Rationale: Oxygen is combustible, so clients should not smoke while using oxygen. The presence of an ashtray should alert the nurse that this may be occurring. Oxygen concentrators may be plugged into surge protectors, and it is recommended to change the filter every 6 months to 1 year based on usage. The use of natural gas for the household heater is not a risk to the client on home oxygen.

The nurse is caring for a client with dementia who wanders throughout the long-term care facility. Which intervention by the nurse would best ensure the safety of the client? Apply an electronic alert wristband. Explain the risk of walking with no assistance. Reorient the client to time, person and place. Administer an antianxiety medication.

Apply an electronic alert wristband. Rationale: A "wandering" management system is used to give clients with dementia and other "at risk" clients the ability to move freely where they live. The sensor in the bracelet sets off an alarm that is attached to exterior doors if the client attempts to leave the facility. Explaining the risks of walking without assistance may not help as the client may not understand or remember the instructions. Reality orientation is inappropriate for someone with dementia. It is inappropriate and unethical to use medications as chemical restraints to stop the client from wandering.

A nurse is providing care to a client with a wound to the left heel. The wound is open and has eschar tissue on the surface. Which prescription from the healthcare provider will the nurse clarify? Apply negative pressure wound therapy Teach toe touch weight bearing to the left lower extremity Irrigate wound twice a day Position with pressure off left lower extremity

Apply negative pressure wound therapy Rationale: Negative pressure wound therapy (NPWT) promotes wound closure and healing. NPWT should not be applied over eschar tissue as it can promote additional bacterial growth. Eschar tissue is dead tissue that should be removed prior to application of NPWT. Toe touch weight bearing is an acceptable mobility prescription for a client with a wound to the heel. Irrigation is an expected treatment to clean an open wound and remove surface bacteria. Pressure offloading is an expected treatment for wounds to the heel area. Decreasing pressure increases circulation to the area and promotes wound healing.

The charge nurse is observing nursing personnel provide client care during an external disaster in the community. Which of the following actions by a nurse would require the charge nurse to intervene? Inserting an oropharyngeal airway in an unconscious client Splinting an injured client's extremity in the position they are found. Applying pressure after removing penetrating objects from clients Providing warming blankets to clients

Applying pressure after removing penetrating objects from clients Rationale: Guidelines for first aid treatment during a disaster include protecting clients' airways, splinting injured parts in the position they are found, maintaining normothermia, and retaining penetrating objects in place. The charge nurse should intervene if staff is observed removing the penetrating objects and applying pressure, as these objects should be left in place; removing them could cause further internal damage and hemorrhage. The other actions are all correct nursing actions during a disaster.

The nurse is caring for a client who is prescribed a new medication. Prior to the administration of the medication, which action by the nurse best demonstrates an awareness of safe and competent nursing practice? Ask the client for their name, date of birth and allergy history, then check the client's identification band. Verify the order for the medication on the admission sheet, then ask the client their name. Ask the client for their medical record number, then check the client's identification band. Verify the client's name on the admission sheet, then check the client's identification band

Ask the client for their name, date of birth and allergy history, then check the client's identification band. Rationale: Safe medication administration is an important nursing skill. Using at least two client identifiers must always be done prior to administering a medication. This involves verbal and visual checks. Because this is a new medication, an allergy check is also appropriate. The nurse demonstrates best practice by asking the client for their name, date of birth and allergy history and then checking the client's identification band. The other actions do not adhere to best practice recommendations for medication administration.

A nurse is preparing to administer intravenous medication to a client. Which action by the nurse ensures the medication is administered to the correct client? Asks the client to state their name and date of birth Verifies the medication has a client label Checks the medication against the prescription and the MAR Ensures the client has an identification wristband

Asks the client to state their name and date of birth Rationale: Proper identification of a client requires two identifiers. The client's name and date of birth are two commonly used identifiers. Verifying the medication has a client label should be performed before dispensing the medication. This process does not ensure it is administered to the correct client. Checking the medication against the prescription and the medication administration record is one of the three checks of medication administration. However, this does not ensure that the medication is administered to the correct client. Ensuring the client has an identification wristband does not guarantee the information printed on the wristband is correct.

The nurse is working with an unlicensed personal assistant (UAP) to care for a client who has soft, bilateral wrist restraints placed. Which task is the responsibility of the nurse? Assess the client's skin underneath the restraints Provide the client with fluids every two hours Assist the client to the bathroom every hour Set up the client's meal tray

Assess the client's skin underneath the restraints Rationale: Restraints are physical devices that can limit the client's movement. The nurse is to protect and promote the client's rights while in restraints. The client should be provided opportunities to use the restroom, drink, and eat, which are tasks that can be delegated to the UAP. The nurse is responsible for the assessment of the client, which includes skin integrity and circulation.

A nurse is assisting a client out of bed. As the client begins to stand, the bed moves back, and the client falls to the ground. The nurse notes the brakes on the bed were not activated. Which priority action does the nurse perform? Informs the charge nurse Assists the client back to bed Locks the brakes on the bed Assesses the client for injury

Assesses the client for injury Rationale: Assisted falls to the ground can still result in client injury. The nurse should assess the client for injuries prior to performing other interventions. Informing the charge nurse is an important action for the coordination of care. However, ensuring client safety is the priority. Assisting the client back to bed should occur after the nurse ensures there are no injuries from the fall. Locking the brakes on the bed will prevent a future fall. However, this action is not the priority.

A nurse is admitting a client after an intravenous pyelogram. The nurse notes an allergy to shellfish on the client's medical record. Which action does the nurse perform next? Provides oral fluids to the client. Assesses the client's airway. Monitors client's urinary output. Checks the client's intravenous line.

Assesses the client's airway. Rationale: An intravenous pyelogram uses contrast media to examine the urinary structures on an x-ray. Contrast media is contraindicated in clients with an allergy to iodine. Clients with an allergy to shellfish may also have a cross allergy to iodine. The nurse should assess the client for signs and symptoms of an allergic reaction. Providing oral fluids and monitoring the client's urinary output are important actions after the procedure. However, these actions are not the priority. Checking the client's intravenous line assesses for patency after contrast media is injected. However, the priority is to assess the client for an allergic reaction to the contrast.

The charge nurse is working on the medical floor when the fire alarm goes off. The fire is on the same floor, but on the other side of the fire doors. Which of the following preparations should the nurse take first? Assign a nurse to be responsible for manning the oxygen shut-off valve Plan to evacuate all clients vertically Remove all equipment from the hallways Implement the R.A.C.E. program

Assign a nurse to be responsible for manning the oxygen shut-off valve Rationale: In the event of an emergency that would potentially allow oxygen to escape into the immediate area or room intensifying a fire hazard, oxygen and medical gas should be shut off. Clients may shelter in place unless there is an immediate threat to health and safety. Equipment should be moved from the hallways in the event of an evacuation, but that is not the priority. Rescue, Alarm, Contain, Extinguish/Evacuate (R.A.C.E.) would have already been implemented at this point.

The nurse is stuck in the hand by an exposed needle that was accidentally left in the client's bed. Which action should the nurse take first? Contact employee or occupational health services. Look up the policy and procedure on needlestick injury. Immediately wash hands vigorously with soap and warm water. Notify the nursing supervisor and complete an incident report.

Immediately wash hands vigorously with soap and warm water. Rationale: The immediate action of vigorously washing the hands will help reduce the risk of potential exposure to bloodborne pathogens. The nurse should then follow the facility's policy and procedure for employee needlestick injury.

The nurse working with an unlicensed personal assistant (UAP) to care for a client who has soft, bilateral wrist restraints placed. Which task should the nurse delegate to the UAP? Evaluate the client's ability to move fingers. Observe the client's capillary refill time. Assist the client during mealtimes. Maintain documentation of restraint criteria.

Assist the client during mealtimes. Rationale: A client who is in soft wrist restraints will require an assessment of skin and circulation and maintain documentation on restraint criteria, which is the responsibility of the nurse. The nurse can delegate to the UAP assisting the client to the bathroom and with eating and drinking.

A nurse is to administer meperidine (Demerol) 100 mg, atropine sulfate 0.4 mg, and promethazine (Phenergan) 50 mg IM to a pre-operative client. Which action should the nurse take first? Assist the client to the bathroom Instruct the client to remain in the bed Raise the side rails on the bed Place the call bell within the client's reach

Assist the client to the bathroom Rationale: Meperidine is a narcotic analgesic and promethazine is an antihistamine; together they can potentiate CNS effects such as drowsiness, dizziness, lightheadedness and confusion. Although all of the options involve client safety, the first thing to do is to assist the client to the bathroom to void. After administering the preoperative medications, the nurse will instruct the client to remain in bed, place the call light in the client's hand and raise the side rails.

The charge nurse working in the pediatrics unit responds to the doorbell of the locked unit. A hospital contractor is at the door to do maintenance but has no identification and is unaccompanied. Which of the following responses is appropriate? Allow the contractor to enter the unit and observe them closely Call security to escort the contractor to the security office for identification verification Ask someone from maintenance to meet the contractor on the unit Direct the contractor to the nurse manager's office to follow up

Call security to escort the contractor to the security office for identification verification Rationale: Identification programs are in place as a form of security planning. This includes employees as well as contractors. In sensitive areas like the pediatrics unit, this is of particular importance in preventing child abduction. In this scenario, it is appropriate to call security to escort the individual to the security office. Someone from the maintenance office can verify the contractor and have an identification tag created. It is important to call security in the event that the contractor is not authorized to be in that location. Failure to notify security may put other staff, visitors, and clients at risk.

A nurse is instructing an unlicensed assistive personnel (UAP) on how to take manual blood pressure on a client. Which action by the nurse indicates correct use of the blood pressure equipment? Selects a cuff with a bladder length that is 60% of the arm's circumference Lines up the arrow on the cuff with the radial artery Closes the valve on the bulb by turning it counterclockwise Centers the deflated cuff 1 inch above the brachial artery

Centers the deflated cuff 1 inch above the brachial artery Rationale: The deflated cuff should be centered 1 inch above the brachial artery to allow room for the stethoscope diaphragm to placed. The length of the cuff bladder should be 80% of the arm's circumference to obtain an accurate measurement. The arrow on the cuff should line up with the brachial artery, not the radial. The valve closes when it is turned clockwise.

The nurse is providing burn prevention and home safety education to parents of small children. Which safety measures should the nurse include in the teaching plan? Select all that apply. Set the water heater temperature to 130° F (54.4° C). Check seatbelt buckles before placing a child in a car seat. Replace smoke detector batteries once per year. Create a fire escape plan and practice it with the family. Turn pot handles towards the center of the stove when cooking

Check seatbelt buckles before placing a child in a car seat. Create a fire escape plan and practice it with the family. Turn pot handles towards the center of the stove when cooking Rationale: Accidents are usually caused by human behavior and error; thus they can be prevented. Nurses must work closely with parents to prevent accidents from occurring in the home. To prevent thermal burns at home, hot water heaters should be set below 120° F (48.8° C). Parents and caregivers should also test the bath water before placing a child in it. When cooking, pot handles should be turned towards the center or back of the stove to prevent an item from being pulled down by a child. A parent or caregiver should check seatbelt buckles before placing a child in a car seat. Buckles can become hot sitting in the sun. Fire escape plans should be practiced with children, and they should know what to do in case of a fire. Smoke detector batteries should be replaced regularly, at least once a year.

The nurse is caring for multiple clients during their shift. Which method(s) would be the best approach to correctly identify each client? Select all that apply. Check the client identification bracelet Compare the client to a labeled photograph Ask clients to state their name Ask a family member or visitor Have clients state their birth date

Check the client identification bracelet Compare the client to a labeled photograph Ask clients to state their name Have clients state their birth date Rationale: The best approaches to correctly identify a client would be to check the client's identification bracelet, ask clients to state their name and date of birth, and compare the client to a labeled photograph. Two pieces of identification are required prior to any procedure and/or medication administration. In long-term care facilities, residents may not wear identification bands. In this case, a labeled photograph can be used as identification. Asking visitors and family members would not be the best approach to identify clients.

The nurse is assessing a client who is 2 days post-operative from coronary artery bypass graft surgery and is sitting up in a chair. The nurse observes that the client is cold to the touch, appears pale, and is reporting dizziness. Which intervention should the nurse take first to prevent injury to the client? Apply oxygen at 2 liters. Check the client's vital signs. Put the client back in bed. Call the healthcare provider.

Check the client's vital signs. Rationale: When caring for a client who becomes dizzy in a chair, the nurse should first assess the client's vital signs. Assessing blood pressure and heart rate can evaluate if the client can safely be moved back to the bed. If the client is hypotensive or bradycardia, the nurse will require additional assistance to safely move the client. The nurse should assess the SP02 before administering oxygen. The nurse will call the healthcare provider once the client is stable.

The nurse is performing triage on clients during a facility disaster drill. Which of the following indicates correct understanding of disaster triage? Classifying a client with a pneumothorax as emergent Classifying a client with a cervical spinal cord injury as urgent Classifying a client with an open femur fracture as non-urgent Classifying a client with a scalp laceration as expectant

Classifying a client with a pneumothorax as emergent Rationale: In mass casualty disaster situations, triage focuses on doing the greatest good for the greatest number of people. Therefore, when resources are severely limited, some clients who have very extensive critical injuries who would otherwise receive massive resuscitation efforts (cervical cord injury, head injuries, massive burns), may be classified as expectant, or black-tagged and allowed to die or not be treated. Clients with airway compromise or shock are classified as emergent and are seen immediately. Clients with open fractures or wounds should be classified as urgent and seen within 30 minutes to 2 hours. Clients with abrasions or contusions should be classified as non-urgent and can be seen after 2 hours or when other more urgent clients have been attended to.

The nurse in the dialysis center suspects that a client receiving hemodialysis is infected with scabies. Which transmission-based precautions should the nurse implement immediately? Contact precautions Neutropenic precautions Bloodborne precautions Airborne precautions

Contact precautions Rationale: Contact precautions reduce the risk of transmission by direct or indirect contact. Indirect transmission involves contact with a contaminated object. Scabies is a parasitic skin infection that is transmitted by direct, physical contact with infected individuals or by sharing clothing or bedding with an infected individual. The other precautions are not appropriate for preventing the transmission of scabies. Bloodborne precautions are not transmission-based precautions. Those precautions fall under standard precautions, which are taken for every client when the possibility of exposure to blood and/or bodily fluids exists, regardless of the presence of a communicable infection.

A client with a diagnosis of methicillin-resistant Staphylococcus aureus (MRSA) has died. Which transmission-based precautions should be used when performing postmortem care? Standard precautions Contact precautions Airborne precautions Droplet precautions

Contact precautions Rationale: MRSA is transmitted by contact and MRSA bacteria remain alive for up to 3 days after a client dies. Therefore, contact precautions must still be used, including the use of a gown and gloves. The body should also be labeled as MRSA contaminated so the funeral home staff can protect themselves as well.

The nurse is assessing a client who is sitting up in a wheelchair and is wearing a safety vest. Which finding by the nurse would require immediate intervention? Nausea reported by client Diminished breath sounds Hyperactive bowel sounds Headache reported by client

Diminished breath sounds Rationale: A safety vest is worn when a client is sitting up in a wheelchair or chair and ensures the client does not slide out of the chair. The vest, which is a type of physical restraint, is positioned over the client's chest with the straps on the back of the vest. If the vest is too tight around the chest, the client is at increase risk of impaired breathing, which could result in diminished breath sounds. If the vest is too tight around the abdomen, the client could report nausea and have hypoactive bowel sounds. A headache is not a complication of wearing a safety vest.

The nurse is preparing a sterile field at the bedside a client with confusion. While opening up supplies, the client reaches over the field and touches several instruments. Which is the appropriate action for the nurse to take? Ask another nurse to hold the hand of the client and continue setting up the field Remove the instruments that were touched by the client and continue setting up the sterile field Discard the supplies and prepare a new sterile field with another person holding the client's hand Apply a soft wrist restraint so the client does not touch the field again

Discard the supplies and prepare a new sterile field with another person holding the client's hand Rationale: Once an item is touched in the field, the whole field is unsterile. The field should be discarded, and a new field should be set up with someone holding the client's hands. You would not continue to set up the field since it is unsterile. Removing the instruments that were touched does not correct the situation that the field is still unsterile. Applying a wrist restraint may increase the confusion of the client and needs a healthcare provider's prescription.

The nurse recognizes that client identification in accordance with agency policy must occur immediately prior to which of the following actions? Select all that apply. Discontinuation of an intravenous normal saline infusion Placement of the call light activation device within reach of the client Collection of a point of care blood glucose test Insertion of an indwelling urinary catheter Administration of oral acetaminophen

Discontinuation of an intravenous normal saline infusion Collection of a point of care blood glucose test Insertion of an indwelling urinary catheter Administration of oral acetaminophen Rationale: As part of safe nursing care, the nurse must collect client identification with at least two approved identifiers according to agency policy immediately prior to medication administration, implementation of health care provider prescriptions, collection of laboratory samples, discontinuation of intravenous infusions and many additional situations. It would not be required to confirm the client's identification immediately prior to placing the call light activation device within reach.

The nurse is caring for a client who lives in a long-term care (LTC) facility. The client is placed on contact precautions when drainage from a wound culture is positive for methicillin-resistant Staphylococcus aureus (MRSA). Which of the interventions should the nurse include in the client's plan of care? Select all that apply. Educate the client on good personal and hand hygiene Monitor staff compliance with using required personal protective equipment (PPE) Collaborate with the facility infection preventionist on treatment for the wound Plan to transfer the client to the hospital Move the client to an available private room Notify the client's family that no visitors are allowed until the infection is cured

Educate the client on good personal and hand hygiene Monitor staff compliance with using required personal protective equipment (PPE) Collaborate with the facility infection preventionist on treatment for the wound Plan to transfer the client to the hospital Move the client to an available private room

A client arrives in the emergency department after a radiological accident at a local factory. After placing the client in the decontamination room, the nurse shall give priority to which intervention? Ensure physiological stability of the client. Double bag the client's contaminated clothing. Wrap the client in blankets to minimize staff contamination. Begin decontamination procedures for the client.

Ensure physiological stability of the client. Rationale: Acute radiation syndrome (ARS) is caused by irradiation of the body by a high dose of radiation in a very short period of time. The treatment goals for ARS are to prevent further radioactive exposure and treat life-threatening injuries. The nurse must initially assist in the stabilization of the client prior to the implementation of any other tasks related to contamination. Evaluating the client's airway, breathing, and circulation, i.e., physiological functions, is the priority. Once the client is deemed stable, the decontamination process can begin. Decontamination involves removing radioactive particles. Removing clothing and shoes (e.g., double bagging clothes) eliminates external contamination.

A client who reports unintended weight loss, drug abuse, and night sweats is admitted with a preliminary diagnosis of HIV/AIDS. Which of these existing clients would be the most appropriate roommate for this newly admitted client? Lupus and vesicles on one side of the middle trunk from the back to the abdomen Acute tuberculosis with a productive cough of discolored sputum for more than three months Pseudomembranous colitis and C. difficile Exacerbation of migrating polyarthritis with severe pain

Exacerbation of migrating polyarthritis with severe pain Rationale: It is most appropriate to place clients with similar diagnoses in the same room. Because this option does not exist, the nurse would understand that a client with HIV/AIDS would be immunocompromised and should not be placed in a room with any client with an active infection. Of the available options, the client with arthritis would be the best roommate for the client with HIV/AIDS. Typically, standard precautions would be used for a person diagnosed with HIV/AIDS (unless the person presents with cough/fever/pulmonary infiltrate, in which case the person would be placed in a private room and airborne plus contact precautions would be implemented).

The nurse smells smoke and notices a small fire in a non-client storage area. The alarm system begins to sound. Which action should the nurse take next? Wait for the arrival of the fire department. Extinguish the fire using an ABC fire extinguisher. Back out of the room and close the door. Place a thermal blanket over the fire.

Extinguish the fire using an ABC fire extinguisher. Rationale: A fire in any health care facility presents great potential for harm. In this situation, there are no clients in imminent danger and the alarm has been activated. The nurse should attempt to extinguish the fire using an appropriate fire extinguisher. The ABC type is appropriate for all types of fires. Backing out of the room and closing the door may allow the fire to burn out of control. Using a blanket is not appropriate at this time. If the fire is manageable, the nurse should attempt to extinguish it and not wait for the fire department to arrive.

A nurse is preparing a client for a 12-lead electrocardiogram. Which anatomical site indicates correct placement of the electrodes? Fourth intercostal space, left sternal border, to obtain lead V3 Left midaxillary line, third intercostal space, to obtain lead V6 Fifth intercostal space, left midclavicular line, to obtain lead V4 Right sternal border, second intercostal space, to obtain lead V1

Fifth intercostal space, left midclavicular line, to obtain lead V4 Rationale: An electrocardiogram monitors the electrical activity of the heart. The placement of leads is important to obtain accurate readings. Lead V4 provides information on the anterior myocardial wall. The electrode should be placed in the 5th intercostal space (ICS), left midclavicular line. The 4th ICS, left sternal border, corresponds with lead V2. Lead V6 requires an electrode to be placed on the left midaxillary line, 5th ICS. Lead V1 is obtained by placing an electrode on the right sternal border, 4th ICS.

The home-health nurse is assessing the client's home for electrical hazards. Which of the following findings requires intervention? Electrical cords are coiled and taped to the ground. Electrical appliances are placed away from wet areas. Frayed electrical cords are taped with electrical tape. Appliances are plugged into grounded outlets.

Frayed electrical cords are taped with electrical tape. Rationale: It requires intervention if the client has repaired frayed electrical cords with electrical tape. All damaged equipment should be evaluated and repaired by an electrician. It is correct for the client to coil and tape cords to the ground, to keep appliances away from wet areas, and to use grounded outlets.

The nurse at an ambulatory care clinic is participating in an active shooter drill. The shooter is rapidly approaching the nurse's location. Which of the following responses is best? Immediately evacuate the building Hide in a room with no windows Attack the shooter at the head and neck Call Emergency Services

Hide in a room with no windows Rationale: The best action in an active shooter situation is to run. However, if the shooter is rapidly approaching the room you are in, the best strategy is to hide as soon as possible. Attacking the shooter is the final option. Emergency Services are notified by those who are not in immediate danger.

The nurse is caring for a child with a diagnosis of suspected pertussis (whooping cough). What is the priority nursing intervention for this child? Implement droplet precautions Initiate anti-infective therapy Monitor respiratory rate and oxygen saturation Maintain hydration and encourage fluid

Implement droplet precautions Rationale: Although all of the responses are appropriate nursing interventions, the priority is to implement strict droplet precautions in addition to standard precautions. Pertussis is highly contagious and is spread through close contact. Therapeutic management focuses on providing respiratory support and eradicating bacterial infection (macrolides, such as erythromycin, are the drug of choice). Administer fluids and keep the client hydrated to help thin secretions. It is also important to monitor the client's heart rate, respiratory status, and oxygen saturation, especially during coughing paroxysms.

The nurse is caring for a client diagnosed with prostate cancer who is prescribed oral cyclophosphamide. Which of the following actions should the nurse take when caring for the client on this medication? Instruct the client to wear gloves when picking up the medication. Crush the medication before administration. Inform the client that any unused medication should be flushed down the toilet. Advise the client to take the medication with food.

Instruct the client to wear gloves when picking up the medication. Rationale: Oral cyclophosphamide is a cytotoxic medication that is used for many cancers. Clients may self-administer these medications at home and, therefore, they may be used outside of an oncology unit. However, this does not mean the drug is safer via the oral route. Gloves should always be worn when handling the medication. If gloves are not worn by the client, teach them to tip tablets and capsules from their container/blister pack directly into a disposable medicine cup. Oral chemotherapeutics should not be crushed. These medications should be disposed of as cytotoxic waste according to the local waste disposal regulatory guidelines. Cytoxan should be taken on an empty stomach unless irritation occurs.

The nurse observes a nursing assistant using antiseptic hand sanitizer and rubbing their hands vigorously after leaving the room of a client diagnosed with Clostridium difficile. Which action by the nurse is appropriate? Instruct the nursing assistant to use bleach wipes to wipe off their hands Report the nursing assistant to the infection control practitioner Praise the nursing assistant for proper use of antiseptic hand sanitizer Instruct the nursing assistant to wash their hands again with soap and water

Instruct the nursing assistant to wash their hands again with soap and water Rationale: Anyone who is hospitalized should be encouraged to ask caregivers if they have washed their hands and should also remind visitors to wash their hands. However, it is the nurse's responsibility to supervise the nursing assistant and to correct practice errors as needed. Clostridium difficile is one of the few pathogens that require soap and water for cleansing the hands. Since antiseptic hand rub is ineffective against the hardy spores produced by this bacterium, the nurse should require the nursing assistant to wash their hands with soap and water, especially after providing care for this client.

A newly admitted client has a skin ulcer that tested positive for methicillin-resistant Staphylococcus aureus. Which precautions should the nurse take when caring for this client? Select all that apply. Keep all equipment in the client's room for their sole use Place the client in a private room. Wear a mask while providing routine care to the client. Perform hand hygiene after contact with the client. Keep the door to the room closed at all times. Place personal protective equipment at the door to the room.

Keep all equipment in the client's room for their sole use Place the client in a private room. Perform hand hygiene after contact with the client. Place personal protective equipment at the door to the room. Rationale: Contact precautions are recommended in acute care settings for methicillin resistant Staphylococcus aureus (MRSA) when there is a risk for transmission or wounds that cannot be contained by dressings. All equipment, such as stethoscopes, should be for the client's sole use and kept in the room. The client should be in a single room or share a room with another client who has MRSA. It is not necessary for the nurse to wear a mask when providing routine care (e.g., collecting vital signs) to the client. Health care workers must perform hand hygiene (e.g., wash hands with soap and water) after direct contact with the client and before leaving the isolation room. Contact precautions require health care workers to wear personal protective equipment (PPE) such as gloves and a gown, which should be readily available. It is not required to keep the door closed at all times. MRSA is not spread by droplet or airborne transmission.

A school nurse is caring for a group of school-age children who have been diagnosed with ringworm (Tinea corporis). The nurse is preparing educational materials for parents on ways to prevent the spread of ringworm. What information should the nurse include? Select all that apply. Keep child at home for 5 to 7 days. Keep fingernails short and clean. Do not walk barefoot in public showers. Change socks daily. Wash hands after playing with pets. Do not share combs or brushes.

Keep fingernails short and clean. Do not walk barefoot in public showers. Change socks daily. Wash hands after playing with pets. Do not share combs or brushes Rationale: Ringworm (Tinea corporis) is a skin and nail fungal infection caused by a parasite that lives in cells on the outer layer of skin. Ringworm can spread through the environment by coming in contact with contaminated surfaces (walking barefoot), through person-to-person contact and by sharing personal items. Ringworm can also spread from an infected pet. Information on how to prevent transmission should include keeping nails short and clean, not sharing personal and grooming items such as combs, brushes, hats, scarves and bedding. The fungus prefers a moist environment. Clothing such as socks and underwear that are in contact with areas of the body that are moisture prone should be changed daily. Isolating the child at home is not necessary.

The nurse is reviewing the plan of care for a client with a head injury who has a prescription for seizure precautions. Which action should the nurse take? Place the bed in high position Keep restraints at the bedside Keep suction equipment at the bedside Remove all pillows from the client's bed

Keep suction equipment at the bedside Rationale: A client with a head injury has an increased risk for seizures. A head injury and seizure can cause airway obstruction and breathing impairment. Oxygen and suctioning may be needed to keep the airway open, so it should be available at the bedside. The bed needs to be in the lowest position possible, and a pillow should be underneath the patient's head to protect it from injury. The use of restraints would not protect the client from injury related to a seizure and could cause musculoskeletal damage.

The nurse is caring for a client with a prescription for seizure precautions. The nurse should implement which action? Apply soft restraints on the client's upper extremities Move the client closer to the nursing station Provide only paper dishes with meals Keep the client's bed in the low position

Keep the client's bed in the low position Maintaining the bed in the low position will decrease the chance of injury from a fall during a seizure. Moving a client closer to the nursing station is for a client who is confused but does not prevent injury from a seizure. Restraints are used to keep a client safe from self-harm. Providing paper dishes is an action related to suicide precautions but not seizures.

The nurse is caring for a client who is receiving internal radiation therapy. Which of the following actions by the nurse is appropriate for reducing radiation exposure? Limit exposure to 60 minutes per day Have visitors keep 6 feet away from the client Wear a gown and gloves when caring for the client Measure radiation omitted using a radiation survey meter

Limit exposure to 60 minutes per day Rationale: The goal is to deliver safe, efficient care in the shortest amount of time. Exposure for the nurse, health care provider, and visitors should be limited to 30 minutes per 8-hour shift. Visitors should keep a distance of at least 6 feet. A gown and gloves would protect against exposure to cytotoxic agents used in chemotherapy, but not radiation. Lead would be used for this purpose. Radiation survey meters are used for disasters involving potential radiation exposure, but not for radiation therapy. Healthcare providers may wear dosimeters to measure exposure.

A client is being assessed for flu-like symptoms and a red bite on the leg. The nurse suspects the client has Lyme disease. Who should the nurse notify? Center for Disease Control and Prevention Local health department Infectious disease specialist Hospital risk management

Local health department Rationale: Lyme disease is a trackable disease reportable to the health department. Upon confirmation, the health department may report it to the CDC. The nurse would not need to report the information to an infectious disease specialist unless the client has received a prescription for treatment. Hospital risk management is notified when there is an ethical or legal issue related to client care.

The nurse is assessing the client's ability to perform proper use of a cane while ambulating. Which of the following actions by the client requires intervention? Stepping with the weaker leg after moving the cane Holding the cane on the client's stronger side of the body Looking down at the cane while moving the legs forward Maintaining the top of the cane at the level of the hip

Looking down at the cane while moving the legs forward Rationale: It indicates proper use of a cane to move the cane first, followed by the weaker leg, and then the stronger leg, which provides a constant wide base of support. The client should hold the cane on the stronger side of the body and maintain the cane at the level of the hip. Clients should be instructed to stand up straight and look forward, not down at the cane, to reduce the risk of falling.

The nurse is assessing the client's ability to perform proper use of crutches while ambulating. Which of the following actions by the client requires intervention? Placing crutches at a 6-inch distance in front of the feet. Maintaining the back in straight alignment and eyes gazing forward. Maintaining the weight of the body on the axilla during movement Placing the crutches 6-inches out laterally on either side of the feet.

Maintaining the weight of the body on the axilla during movement Rationale: Correct technique for crutch walking includes placing the crutches at a 6-inch distance both in front of the feet as well as 6 inches out laterally on either side of the feet to maintain a wide base of support. The client should maintain the back in straight alignment and look forward, not down, while ambulating to reduce the risk of falls. It requires intervention if the client is maintaining the weight of the body on the axilla instead of the wrists, as this can cause radial nerve injury and eventually crutch palsy, causing weakness of the muscles of the forearm, wrist, and hand.

A nurse is reviewing prescriptions for a client with a deep vein thrombosis to the right lower extremity (RLE). Which prescription should the nurse clarify? Apply graduated compression stocking to the RLE Massage the RLE as needed Perform range of motion exercises to the RLE Elevate the RLE above the level of the heart

Massage the RLE as needed Rationale: A deep vein thrombosis is a blood clot that forms within a vein and obstructs blood flow. The goal of treatment is to increase circulation to the extremity once anticoagulant therapy is initiated. Massaging the affected extremity is contraindicated, as it can dislodge the blood clot and cause obstruction of other vessels. Applying graduated compression stockings is an expected treatment. Compression stockings increase the velocity of blood flow and improve valve function in the veins. Range of motion exercises are encouraged and improve muscle tone and circulation to the extremity. Elevating the affected extremity above the level of the heart increases venous blood flow.

A nurse is admitting a client from the emergency department diagnosed with an ischemic stroke. The client has right hemiplegia and facial drooping. Which admitting prescription should the nurse clarify? Continuous pulse oximetry Mechanical soft diet Bed rest Neurological checks every hour

Mechanical soft diet A client with an ischemic stroke and visible signs of paralysis is at risk for dysphagia. Clients should be evaluated by a speech-language pathologist (SLP) prior to feeding to avoid aspiration. Continuous pulse oximetry is indicated for clients with an ischemic stroke to monitor oxygenation. Bed rest promotes safety until the client can be evaluated by physical therapy. Frequent neurological checks are expected for a client with an ischemic stroke. The nurse should notify the healthcare provider of any changes.

The nurse is caring for a client with an inner ear infection who is experiencing vertigo. Which intervention would protect the client from injury? Monitor the client for falls Document vital signs Speak to the client on the unaffected side Provide medication for nausea

Monitor the client for falls Rationale: Some clients may experience vertigo, a sensation of spinning, with inner ear infections. Clients with vertigo will report feeling dizzy, which could lead to falls and cause injury. Monitoring for falls will protect the client from injury. Vertigo can also cause nausea and is treated with medication but does not cause injury. Clients may also have hearing loss, which is not an injury, related to vertigo, so speaking to the client on the unaffected side will make sure the client understands the instructions. Documenting vital signs will allow assessing of trends with vertigo but will not prevent injury.

The nurse incorrectly administers carvedilol to a client with an order for benztropine. What is the priority nursing intervention after making this medication error? Complete an incident report Alert the nurse manager Notify the health care provider Monitor the client's blood pressure

Monitor the client's blood pressure Rationale: Because the nurse mistakenly administered a beta blocker medication, the priority intervention is to monitor the client for any adverse physiological response to the given drug. Carvedilol blocks alpha1 and beta receptors in blood vessels, causing dilation and a decrease in blood pressure.

Sputum culture results for a client admitted with a cough and fever indicate a methicillin-resistant Staphylococcus aureus (MRSA) infection in the nares. What nursing intervention must now be taken? Select all that apply. Move the client to a private room Dedicate the use of personal and noncritical medical equipment to the client Place the client in a room with another client colonized with MRSA Place a mask on the client if the client needs to leave the room Staff will wear N-99 or N-100 particulate respirators when in the client's room

Move the client to a private room Dedicate the use of personal and noncritical medical equipment to the client Place the client in a room with another client colonized with MRSA Place a mask on the client if the client needs to leave the room Rationale: When possible, a private room would be best, but cohorting is often used for multidrug-resistant organisms, such as MRSA. If the client needs to be transported to another area, the client should wear a mask, especially if there's a productive cough. Staff should practice excellent hand hygiene and other standard precautions, but a respirator is not needed for MRSA in the nares. To minimize the risk of spreading infection, equipment or personal items should be kept in the client's room and dedicated for his/her use.

The nurse hears a scream coming from a client's room. When entering the room, the nurse finds the client lying on the floor beside the bed. Which of the following actions should the nurse take? Select all that apply. Report the incident to the facility's lawyer. Place the client in physical restraints to prevent another fall. Notify the client's provider about the incident. Take the client's vital signs. Observe the client for abnormal leg rotation. Determine the client's level of consciousness.

Notify the client's provider about the incident. Take the client's vital signs. Observe the client for abnormal leg rotation. Determine the client's level of consciousness. Rationale: Fall prevention is a national patient safety goal and is monitored closely in all health care settings. It is important for the nurse to assess and evaluate the client to determine if the client experienced a loss of consciousness or a change in vital signs that contributed to the fall. It is important to determine if there are visible injuries and note any areas of pain or abnormal leg rotation. The nurse will notify the provider and complete an incident report. Risk management will receive notification through the completion of an incident report; the nurse should not notify the legal department by themselves. Physical restraints are not indicated and may, in fact, make the client more prone for future falls.

A nurse is providing care to a client with hepatorenal syndrome post-dialysis. Upon assessment, the client is confused and the nurse notes jaundice and ascites. Which admitting prescription will the nurse clarify? Implement fall precautions Obtain consents for paracentesis Initiate continuous bedside cardiac monitoring Collect type and crossmatch for packed red blood cells

Obtain consents for paracentesis Rationale: The nurse should clarify the prescription to obtain consents for a paracentesis. Clients who are post-dialysis are at a high risk for bleeding for the first several hours. A paracentesis is an invasive procedure that can cause bleeding. Implementing fall precautions is indicated for a client who is confused. Continuous cardiac monitoring is not an invasive procedure or contraindicated with dialysis. A type and crossmatch is a laboratory test used to assess compatibility of blood for a transfusion. Obtaining labs is not contraindicated post dialysis.

The nurse is caring for a client who is confused and is directly harming staff and self. The nurse applies mitten restraints. Which action is the priority for the nurse to take? Document use of restraints. Obtain prescription for restraints from healthcare provider. Educate family on use of restraints. Notify the charge nurse that restraints have been initiated.

Obtain prescription for restraints from healthcare provider. Rationale: Restraints, which are safety devices that restrict a client's movement, should be implemented as a last resort. Restraints are used when the client's actions could cause harm to self or others. Nurses can apply restraints in emergency situations but are required to obtain a prescription from the healthcare provider immediately after application. Once a prescription is obtained, then the nurse will document, educate family, and notify staff of restraint use.

The nurse is teaching the parents of a toddler-age client about protective measures to reduce injuries at home. Which of the following actions, if identified by the parent, indicates the need for further teaching? Placing covers over electrical outlets Installing screens on windows and balconies Obtaining a bed with high railings Keeping plastic bags stored in a secured container

Obtaining a bed with high railings Rationale: It indicates the need for further teaching if the parent states that they will obtain a bed with high railings for toddlers. Toddlers are prone to climbing and it is recommended to obtain a low bed without railings at this age to prevent falls and climbing-related injuries. It is the correct understanding of protective measures to place covers over electrical outlets, to install screens on windows and balconies to prevent falls, and to keep plastic bags in a secure storage container out of reach to prevent suffocation.

The nurse is responding to a fire on the unit. Which of the following actions by the nurse requires intervention? Opening all the doors and windows on the unit Discontinuing oxygen for all clients who can breathe without it Directing ambulatory clients to walk to a safe location Moving bedridden clients by stretcher to a safe area

Opening all the doors and windows on the unit Rationale: It requires intervention if the nurse is observed opening the doors and windows to allow smoke to escape. Proper response during a fire is to confine the fire by closing doors and windows to reduce the amount of oxygen feeding the fire. Oxygen is flammable, therefore all clients who can breathe without oxygen should have their oxygen removed. Nurses should direct all ambulatory clients to walk to a safe location and focus on moving bedridden clients either in their bed, on a stretcher, or carrying them with two staff on a blanket.

The nurse is preparing a sterile field for a bedside procedure. Which action should the nurse take? Leaving the room after the sterile field is set up for less than 1 minute to check orders Opening sterile supplies away from the body and above the waist Putting on sterile gloves by touching the inside of both gloves Touching the sterile field within 2 inches of the outer edge

Opening sterile supplies away from the body and above the waist Rationale: Opening sterile supplies and/or instruments away from their bodies ensures that the components of the sterile field remain sterile. Nurses should never leave the sterile field unattended, as this automatically deems the field contaminated. Sterile gloves should be applied by only touching the inside of the first hand to don the glove, followed by using the newly sterile gloved hand to only touch the outside of the second glove to apply. Nurses should not touch the sterile field except the outer 1-inch edges of that field while considered unsterile.

The charge nurse is observing a newly hired nurse respond to a fire in a client's room. Which of the following actions by the newly hired nurse requires intervention? Evacuating clients who are in immediate danger Using a fire extinguisher with a sweeping motion at the base of the fire Opening the doors to client rooms and hallways to allow smoke to escape Reporting fire details and location to the agency's emergency response system

Opening the doors to client rooms and hallways to allow smoke to escape Rationale: Correct response during a fire is to use the RACE protocol: rescue, alarm, confine, and extinguish. It is correct for the nurse to evacuate clients in immediate danger, report details of the fire to the emergency response system, and use the fire extinguisher if the fire is small enough to extinguish. The nurse should not open doors to client rooms and hallways; the correct action is to close the doors in order to confine the fire.

A nurse is caring a client who was admitted for diarrhea and has been diagnosed with a Clostridium difficile infection (CDI). To reduce the risk of transmission, which action should the nurse take? Perform frequent hand hygiene with alcohol- based hand sanitizer Clean shared equipment with standard disinfecting wipes Move the client to a negative pressure room Place dirty linens directly into the soiled linen cart

Place dirty linens directly into the soiled linen cart Rationale: C. diff is a spore-forming, gram-positive anaerobic bacillus. It is a common cause of antibiotic-associated diarrhea (AAD). C. diff is shed in feces so any surface, device, or material that becomes contaminated with feces could serve as a reservoir. C. diff spores can also be transferred to patients via the hands of healthcare personnel who have touched a contaminated surface or item. Use contact precautions for patients with CDI. Because alcohol does not kill C. diff spores, the use of soap and water is more effective than alcohol-based hand hygiene. Bleach-based wipes should be used for cleaning surfaces and shared equipment. The linens of a client with C. diff diarrhea may be contaminated and should be immediately placed in the soiled linen cart. Do not lay dirty linens on any surface.

The school nurse is providing information for teachers at a school attended by a 10-year-old child with epilepsy. Which action should a teacher take first in the event the child experiences a seizure in the classroom? Place something soft and flat under the child's head. Notify the child's parent(s) or guardian of the episode. Administer a prescribed antiepileptic medication. Provide privacy and reassurance to the other children

Place something soft and flat under the child's head. Rationale: During seizure activity, the priority is to protect the child from physical injury. Placing something soft and flat, like a folded jacket, under the child's head to help prevent head trauma should be done first. After protecting the head, the next actions should be to move any furniture away from the child and notify the child's parent(s) or guardian. Reassuring the other children is important but should be done after caring for the seizing child first. A teacher should not administer medication, only the nurse is able to do that.

The nurse is caring for a group of clients on a medical-surgical unit. The nurse understands that which situations would require hand hygiene? Select all that apply. Prior to entering a client's room After making an entry in a client's medical record After wiping down a client's bedside table Before and after taking a break and eating lunch After changing a client's sterile dressing

Prior to entering a client's room After wiping down a client's bedside table Before and after taking a break and eating lunch After changing a client's sterile dressing Rationale: Hand hygiene is the most effective way to reduce the transmission of microorganisms and prevent the spread of infection. Hand hygiene (e.g., cleaning hands using soap and water or an antiseptic hand rub) is easy and inexpensive. Additionally, hand hygiene reduces the incidence of healthcare-associated infections (HAIs), as well as the incidence of antimicrobial resistance. It is necessary for the nurse to wash their hands prior to eating, after removing gloves, following any client procedure, and even after having contact with intact skin or objects in the client's room (e.g., the client's bedside table). However, it is not necessary to wash hands after handling the client's medical record.

The nurse is caring for a client who is undergoing internal radiation therapy. Which of the following actions by the nurse is appropriate to ensure safety? Encouraging the client to ambulate in the hallway Following airborne isolation precautions while in the client's room Discarding any dislodged implants into a sharps container Prohibiting visitation from children

Prohibiting visitation from children Rationale: For the client undergoing internal radiation therapy, the nurse should prohibit visitation from children or pregnant individuals. The client should remain in their own private room during ambulation. Airborne precautions are not necessary, but radiation shielding protective equipment should be used. If an implant is dislodged, it should be discarded into a radiation safe container; sharps containers do not protect from radiation.

A nurse is assisting a respiratory therapist with tracheal suctioning on a client. The nurse observes the respiratory therapist break sterile technique multiple times when preparing the equipment. What initial action does the nurse take? Provide the respiratory therapist with a new suction kit Tell the respiratory therapist that sterile technique was breached Document the respiratory therapist's error in the client's record. Inform the respiratory therapist's supervisor about the incident

Provide the respiratory therapist with a new suction kit Rationale: A break in sterile technique is considered negligent care. The nurse should provide the respiratory therapist with a new suction kit to prevent client harm. Telling the respiratory therapist that the sterile technique was breached should be discussed outside of the client's room. Documentation of a procedural error should not be part of the client's medical record. Informing the respiratory therapist's supervisor about the incident is indicated for quality improvement purposes. However, this action should occur after the nurse verifies infection control during the procedure.

The nurse in a long-term care facility is evaluating the plan of care for an older adult client with advanced dementia. The client has had several falls out of bed. Which initial intervention should the nurse implement? Position all side rails of the bed up and move the bed close to the door Put the bed in the lowest position with a thick pad or mat on the floor next to the bed Have the client sleep in a recliner at the nurse's station with a tray table across their lap Place the client in a bed with an enclosure mesh tent attached to the frame

Put the bed in the lowest position with a thick pad or mat on the floor next to the bed Rationale: Falls out of bed are a common occurrence in the long-term care setting. Although it is nearly impossible to eliminate all falls, the nurse can implement interventions to reduce the risk for injury related to a fall. The goal is to start with the least invasive and restrictive intervention to preserve the client's rights, regardless of their level of cognitive function. 'Low' beds and 'landing' mats to soften the fall should the client roll out of bed are commonly used in long-term care settings and represent an appropriate, initial intervention to implement for this client. The other interventions are much more restrictive and should be used only after less restrictive interventions have been attempted.

A nurse is assigned a client with Parkinson's disease experiencing delirium. Which room is best indicated for this client? A room adjacent to an ante room A room facing the elevator A room with multiple windows A semi-private room

Rationale: Clients with Parkinson's may experience neurological changes leading to delirium. Delirium results in impaired judgment, the inability to focus, and disorientation. A room with multiple windows helps the client with time orientation. A room with an adjacent ante room further isolates the client. Ante rooms are indicated for clients with organism precautions. A room facing the elevator and a semi-private room will produce too much noise and visual stimuli for a client with delirium

The nurse receives a telephone call from a health care provider who wants to give a telephone order. Which of the following actions should the nurse take? Select all that apply. Request that the order is signed by the provider before implementation Begin the order with the abbreviation "P.O." to indicate that it was a "phone order" Record the order word-for-word and sign the order Ask a second nurse to listen on another extension while the order is being given Verify understanding by reading the order back to the provider before hanging up

Record the order word-for-word and sign the order Verify understanding by reading the order back to the provider before hanging up Rationale: Reading the order back allows the provider to correct any misunderstanding and is a Joint Commission read-back requirement. The order should be immediately written and signed by the nurse. The order should clearly state "telephone order" as abbreviations can be misunderstood (P.O. could be interpreted as "by mouth"). Having a second person listen in on the conversation is not required unless the nurse cannot understand the health care provider. The order may be implemented right away, but it must be countersigned within the time limits set by the facility.

The home health nurse is preparing for a home visit of a new client. Which action is most important to ensure the safety of the nurse during the visit? Carry a cell phone, pager and/or hand-held alarm. Observe for evidence of weapons in the home Remain alert and leave if cues suggest the home is not safe. Review documentation for previous entries about violence

Remain alert and leave if cues suggest the home is not safe. Rationale: Nurses need to assess and manage safety risks and have ongoing clinical supervision and support when making home visits. The most important action a nurse can take to ensure safety during a home visit is to always remain alert and to leave if there are any cues that the home is not safe. Proper safety should begin with a thorough assessment of the client's home to identify potential risks, such as pets (a commonly assessed hazard), drug use and weapons. The nurse should also develop a plan to eliminate the risks and understand that there is always the option to end a visit early if the environment does not seem safe. Carrying a phone, using a buddy system, learning about the client prior to the visit can also help mitigate risks.

The nurse is obtaining vital signs on a stable client using an electronic blood pressure cuff. The cuff begins to inflate, but the machine is making an unusual sound. The client states that it is painfully tight. Which of the following actions is appropriate? Remove the cuff from the client and take note of which device is defective Remove the device from service and call biomedical services to provide maintenance Obtain the blood pressure with the device and then report the issue to the charge nurse Ask another nurse to assist in troubleshooting the device and obtain the blood pressure

Remove the device from service and call biomedical services to provide maintenance Rationale: Obtaining accurate vital signs is essential to quality nursing care. When a BP machine is not operating correctly, the data it provides may not be reliable and there is a risk of harm to the client if any intervention is based on an inaccurate BP. The device should be removed from service until it can be checked for accuracy and safety and, if needed, repaired.

The nurse is planning care for a 3-month-old infant who needs a cleft lip and soft palate repair. During the immediate postoperative phase, the nurse should give priority to which intervention? Position the infant on the side or back and assess the skin. Provide the infant's family with instructions about care of the sutures. Initiate clear liquid feedings by bottle when alert and acting hungry. Remove the soft elbow/arm restraints every 2 hours and assess the infant.

Remove the soft elbow/arm restraints every 2 hours and assess the infant. Rationale: The priority intervention after surgery is to protect the new repair and stitches, by applying soft elbow and arm restraints. These restraints are used to prevent the infant from putting their hands in their mouth risking injury to the suture line. The nurse should also be assessing for circulation, movement, and sensation every two hours. When the infant acts hungry, they will be given a clear liquid feeding using either a syringe fitted with a special soft tubing or a special cleft lip feeder. The infant should be repositioned on their side and back to prevent skin breakdown; however, another choice is higher priority. Before the infant is dismissed, the nurse will want to provide education to the family regarding the care of the sutures and need to wear the soft restraints for the first 10 days after the surgery.

The nurse is caring for an 80-year-old client with community acquired pneumonia. Since admission, the client has been confused, pulling on tubes including his oxygen mask and peripheral venous access device. The client keeps trying to get out of bed unassisted and has had several near falls. Which interventions should the nurse include in the client's plan of care? Select all that apply. Request an order for a sedative medication at bedtime Place a protective sleeve or elastic bandage over the peripheral venous access device Request a PRN order for restraints Request electronic or in-person client safety monitoring Provide frequent reorientation to the environment Discontinue the oxygen since the client has a Do Not Resuscitate order

Request an order for a sedative medication at bedtime Place a protective sleeve or elastic bandage over the peripheral venous access device Request electronic or in-person client safety monitoring Provide frequent reorientation to the environment Rationale: Older adult clients are at risk for developing delirium when they experience an acute infection and require hospitalization. The unfamiliar environment will contribute to any anxiety, fear and disorientation. It is challenging to care for clients with delirium and the nurse shall implement interventions that preserve the client's dignity and rights while also maintaining the client's safety and ensuring that the client receives all ordered medical therapies. A calm, soothing approach will help to establish trust and although not ideal, a mild sedative to facilitate sleep can be helpful. Many hospitals today utilize electronic or in-person client safety monitors to prevent falls and injury. The client safety monitor remains with the client at all times and verbally reorients the client and provides reminders for the client to wait for assistance. Oxygen therapy should be continued, regardless of resuscitation status, and restraints should never be ordered as needed or PRN.

The nurse is teaching the parents of an infant client about protective measures to reduce injuries at home. Which of the following actions, if identified by the parent, indicates the need for further teaching? Placing covers over electrical outlets Using guard gates on stairs and windows Setting the water heater maximum temperature to 140°F Maintaining the crib slats no greater than 2.5 inches apart

Setting the water heater maximum temperature to 140°F Rationale: It requires further teaching if the parent states that the water temperature maximum should be set at 140°F. It is recommended that the temperature maximum be set at 120°F to minimize the risk of scalding burns during baths. It is the correct understanding of protective measures to place covers over electrical outlets to reduce risk of electrical shock, to use guard gates on stairs and windows to reduce risk of falling, and to maintain crib slats no greater than 2.5 inches apart.

The nurse is assisting a colleague to change the linens on an occupied bed. The client is dependent after experiencing a large cerebrovascular accident. Which of the following actions reduces the risk of injury to the client and staff? Roll the client toward the nurse using the hip and shoulder as hand positions Turn the client away from the nurse using the hip and shoulder as hand positions Push the client to the opposite side using force against the hip and shoulder Drag the client toward the nurse using the slide sheet and shoulder

Roll the client toward the nurse using the hip and shoulder as hand positions Rationale: When lifting any object, keep it close to the body. Therefore, it is best to roll the client toward the nurse who is supporting the weight. Pushing and pulling the client can cause injury to both the nurse and client.

The nurse is preparing to apply soft physical restraints on a client who is attempting to pull out the placed indwelling urinary catheter. Which action should the nurse take? Ensure that the strap is tightly secured around the wrist Flex the client's arms slightly before securing the ties Create a knot in the middle of the ties Secure ties to the bed frame

Secure ties to the bed frame Rationale: Restraints are physical devices that can limit the client's movement and are used to keep the client safe from injury. When applying soft wrist restraints, the nurse should ensure that the strap fits snugly around the wrist, allowing two fingers to be inserted under the strap. If the strap is too tight, it can cause impaired circulation. The nurse should position the client's arms in a natural position to the client's side. Some restraints require the ties to be tied in a knot at the wrist strap, but the ties should be separate through the length. The nurse will secure the ties to the bed frame using a quick-release knot.

A nurse at a community health clinic is speaking to a group of young adults about preventing HIV infection. Which high-risk behaviors to avoid should the nurse include? Select all that apply. Donating blood Sharing needles Smoking e-cigarettes Having unprotected sex Inhaling illegal drugs

Sharing needles Having unprotected sex Rationale: Risk factors associated with HIV include sharing injection drug equipment, having multiple sexual partners, having sexual relations with infected persons, being born to mothers with HIV infection, and not using some form of protection during sex. The nurse should provide preventative education regarding using safer sex practices to reduce the risk of transmitting HIV and avoid sharing any type of needles, razors, toothbrushes, or anything that is potentially contaminated with blood. The other behaviors are not known to increase the risk of contracting HIV.

The nurse is preparing a client who is non-verbal for a lumbar puncture. The nurse instructs the client to remain still during the procedure. Which information should the nurse provide to the client on how to communicate with staff during the procedure? Shake their head back and forth Raise hand above head Squeeze the nurse's hand Use a finger to tap on the bed

Squeeze the nurse's hand Rationale: Clients are instructed not to move during the procedure. Any movement can cause the needle to shift. The nurse helps keep the client in the proper alignment and provides support. Squeezing the nurse's hand requires little movement by the client; the other methods require significant movement.

A client's wound has tested positive for methicillin-resistant Staphylococcus aureus (MRSA). Which transmission-based precautions should the nurse implement for the client? Contact precaution and droplet precautions Contact precaution and airborne precautions Standard precaution and airborne precautions Standard precaution and contact precautions

Standard precaution and contact precautions Rationale: Standard precautions are used for all clients, regardless of their diagnosis or presumed infection status. Transmission-based precautions provide additional precautions beyond standard precautions to prevent transmissions of pathogens. Contact precautions are used for infections such as MRSA that spread by skin-to-skin contact or contact with other surfaces.

Several victims from a mass casualty event are brought to the emergency department. Which client should the nurse see first? The client with multiple wounds and an open fracture The client with hypotension and a sucking chest wound The client with head trauma and agonal respirations The client with a nondisplaced fracture of the radius

The client with hypotension and a sucking chest wound Rationale: The nurse should use the disaster triage color system in combination with the Airway-Breathing-Circulation prioritization approach to decide which client to see first. The client with head trauma and agonal respirations is most likely suffering from brainstem trauma/pressure and seems to be near death; therefore, the nurse should see the client with a sucking chest wound first because that client has a better chance of survival.

The public health nurse is teaching parents about injury and accident prevention in children younger than 4 years old. Which interventions should the nurse include? Select all that apply. Store any firearms in a locked container with the ammunition removed Make sure the child is up-to-date on all required immunizations Store medications where the child cannot reach them Monitor the child for signs of substance use such as alcohol or smoking Never leave the child unattended around a pool or body of water Always have the child sitting in an approved car safety seat when driving

Store any firearms in a locked container with the ammunition removed Store medications where the child cannot reach them Never leave the child unattended around a pool or body of water Always have the child sitting in an approved car safety seat when driving Rationale: Injuries are the most common cause of death and disability to children in the United States. The child's developmental stage partially determines the types of injuries that are most likely to occur at a specific age and helps provide clues to preventive measures. Drowning and burns are among the top three leading causes of death for males and females throughout childhood. In addition, improper use of firearms is a major cause of death among males. Every year, approximately 95% of medication-related ED visits in children younger than 5 years are due to ingesting medication while unsupervised. Motor vehicle injuries are the leading cause of death in children older than 1 year of age. The majority of fatalities involve occupants who are unrestrained.

The nurse is providing an in-service to nursing staff on oxygen safety. Which information should the nurse include in the teaching? Place the cylinder between the client's legs during transport Secure the cylinder in a holder when brought into the MRI suite Store unused cylinders upright in a rack Store empty cylinders on their side

Store unused cylinders upright in a rack Rationale: Compressed gas cylinders must always be in an upright position and supported, whether full or empty. Acceptable methods of support include wall-mounted or bench-mounted gas cylinder brackets, chains or belts anchored to walls or benches, and free-standing dollies or carts designed for gas cylinders and equipped with safety chains or belts. Do not bring oxygen cylinders into an MRI suite as they are ferromagnetic. If a cylinder falls off the bed, it can become a projectile.

Which action by the nurse demonstrates understanding of precautions for an immunocompromised client? Takes the meal tray into the client's room Takes the client's vital signs including the temperature every shift Makes the client wear a mask while care is provided in the room Makes the client brush their teeth before each meal

Takes the meal tray into the client's room Rationale: The nurse taking the tray into the room will limit the number of people entering the room which reduces the exposure to pathogens. Vital signs, including temperature, should be taken every 4 hours to detect potential infection. The client does not need to wear a mask while in their own room but should wear one when leaving the room. Clients should maintain oral hygiene but brushing before each meal is not necessary.

The nurse receives report on the following four clients. Which client should the nurse assess first due to a high risk of falling? The 81 year-old who fell at home last week who has altered mental status The 79 year-old who has rheumatoid arthritis and walks with the aid of a walker The 59 year-old who had hip replacement surgery four days ago and is going to physical therapy The 67 year-old who has diabetes and has chronic draining ulcers on the right leg

The 81 year-old who fell at home last week who has altered mental status Rationale: Although all of the individuals might be at risk for falling, evidence shows that the greatest risk of falling is a person who is older than age 80, is confused, and has a history of falling.

The nurse and UAP are preparing to reposition a client in bed. Which of the following actions indicate that the UAP requires additional training on correct body mechanics? The UAP lifts the client, using their upper arm and shoulder strength. The UAP stands with their feet shoulder width apart and knees slightly bent. The UAP avoids twisting at the hip while repositioning the client. The UAP elevates the height of the bed to about waist level

The UAP lifts the client, using their upper arm and shoulder strength. Rationale: It is important to use proper body mechanics when transferring, lifting or repositioning clients. To apply proper mechanics, the health care worker should stand with their feet shoulder width apart and their knees slightly bent and avoid twisting when repositioning the client. Elevate the bed so the working surface is at waist level, which is the health care worker's center of gravity. The nurse or UAP should bend from the knees to aid in the lift, rather than using their arms and shoulders to reposition the client.

The nurse is planning care for a group of assigned clients. Which client would be most appropriate for the nurse to request a prescription for physical restraints? The client with a history of falls who uses a walker with ambulation The client with dementia who keeps pulling on the inserted nasogastric tube The client with a decrease in visual acuity who is touching the IV site The client with incontinence who is attempting to get out of bed

The client with dementia who keeps pulling on the inserted nasogastric tube Restraints are physical devices that can limit the client's movement and are used to keep the client safe from injury. The nurse should attempt to use alternatives to restraints, such as provide the client with a bedside commode, bed alarms for fall risks, and communicating with the client about interventions. Restraints are used when alternative interventions are unable to keep the client safe from harm, such as to maintain placed tubes or prevent injury to staff.

The nurse is caring for an 80-year-old client who requires wrist restraints. Which client behaviors would support the need to continue to use restraints? Select all that apply. The client is directing frequent angry, verbal outbursts at the staff. The client is confused and trying to pull out an IV catheter. The client is trying to get out of bed without assistance. The client is resisting care and attempting to hit the staff. The client is pushing the call light button every 10 minutes.

The client is confused and trying to pull out an IV catheter. The client is resisting care and attempting to hit the staff. Rationale: Physical restraints should only be used as a last resort. If restraints are indicated, the least restrictive device available should be used to restrain the client. The restraint should protect the individual but also allow for freedom of movement. Circumstances that require the use of physical restraints include when clients attempt to remove life-support equipment, when clients interfere with therapy or treatment (e.g., enteral feedings, intravenous infusions, tracheostomy tubes, etc.), and when clients are combative and a risk to others. Restraints are not indicated for the convenience of hospital staff. Examples of physical restraints include hand mitts, arm sleeves, lap belts, and limb restraints.

The nurse is caring for a client with schizophrenia, who has an order for haloperidol 5 mg PO every four hours as needed. Which behaviors justify the use of this chemical restraint? Select all that apply. The client is expressing paranoid delusions. The client is verbalizing a plan to harm another client. The client is refusing to participate in unit group activities. The client is experiencing command hallucinations. The client is crying after a difficult family meeting.

The client is expressing paranoid delusions. The client is verbalizing a plan to harm another client. The client is experiencing command hallucinations. Rationale: Command hallucinations and paranoid delusions can be frightening or dangerous, potentially causing a client to act aggressively. It is important to intervene before a client act on a plan to harm another person. An antipsychotic medication, such as haloperidol, will help control and manage symptoms and behaviors associated with schizophrenia. A chemical restraint should be used in an extreme or emergent situation. A client has the right to refuse to participate in activities. Verbal intervention, such as offering to speak with the client 1:1, would be appropriate if the client is upset and crying.

The nurse is triaging clients during an external disaster. Which of the following clients should the nurse see first? The client who has a respiratory rate of 36 and has asymmetrical chest expansion The client who has full-thickness burns extending over 75% of the body The client who has an open femur fracture with palpable distal pulses The client with a scalp laceration who has a moderate amount of bleeding from the site

The client who has a respiratory rate of 36 and has asymmetrical chest expansion In mass casualty disaster situations, triage focuses on doing the greatest good for the greatest number of people. Therefore, when resources are severely limited, some clients who have very extensive critical injuries who would otherwise receive massive resuscitation efforts (cervical cord injury, head injuries, massive burns), may be classified as expectant, or black-tagged and allowed to die or not be treated. Clients with airway compromise or shock are classified as emergent and seen immediately, such as the client who is exhibiting a potential pneumothorax. Clients with open fractures or wounds should be classified as urgent and seen within 30 minutes to 2 hours. Clients with abrasions or contusions should be classified as non-urgent and can be seen after 2 hours or when other more urgent clients have been attended to.

The nurse is participating in a disaster drill in which clients must be discharged to make room for unstable victims. Which of the following clients would require intervention by the nurse? The client who is ambulatory and was admitted 2 days ago for observation for lightheadedness and palpitations The client who requires assistance with a walker and is awaiting diagnostic testing for possible lung cancer The client who is on bed rest and had a cardiac catheterization 2 hours ago for evaluation of chest pain. The client who is awaiting transfer to a rehabilitation facility following a minor stroke

The client who is on bed rest and had a cardiac catheterization 2 hours ago for evaluation of chest pain. It may be necessary during disaster situations for nurses to determine clients who are stable enough for discharge to make room for unstable victims. Clients who should be considered for discharge include those who are admitted for observation and are not bedridden, those who are having diagnostic evaluations and are not bedridden, and those who could be cared for in another health care facility, such as long-term care or rehabilitation. The client who just underwent cardiac catheterization for chest pain must be monitored closely for complications related to the procedure and the workup for chest pain and is not suitable for discharge.

The nurse is completing a home health visit for a client who has limited mobility. Which of the following findings indicates the need for intervention to ensure safety? The client's bedroom is on the second floor of the home Grab bars are present in the client's shower The client uses a walker to ambulate in the home Exterior windows have a locking mechanism

The client's bedroom is on the second floor of the home Rationale: If the client has limited mobility, a second-floor bedroom is not ideal. Having to go up and down stairs increases the risk of falls. If the client must have a bedroom on an upstairs floor, modifications and education are imperative to ensure safety. Grab bars and walkers both are devices that can assist the client to perform the activity safely. Locking exterior windows does not increase the risk of injury.

The nurse is preparing to enter a disaster scene to assist with triaging victims. What assessment priorities should the nurse adhere to? Select all that apply. The nurse should consider the age of a victim before allocating any resources. The nurse must consult a qualified health care provider prior to making client resource decisions. The nurse requires disaster certification before performing triage during a disaster. The nurse should assess clients by considering their airway, breathing, circulation and neurological function. The nurse should allocate resources to those victims with the strongest probability of survival

The nurse should assess clients by considering their airway, breathing, circulation and neurological function. The nurse should allocate resources to those victims with the strongest probability of survival Rationale: The goal of disaster triage is to use resources for clients with the strongest probability of survival. Age is not a consideration when allocating treatment resources and the nurse does not need to consult a physician prior to making decisions about allocating resources. Furthermore, a nurse does not need special training to assist in a disaster. However, there are certifications available for nurses who are interested. Finally, the nurse will make decisions based on a client's airway, breathing, circulation and neurological function.

While working a 12-hour night shift, the nurse has a "near miss" and catches an error before administering a new medication to the client. Which factors could have contributed to the near miss? Select all that apply. The nurse works in the intensive care unit (ICU) The nurse was interrupted when preparing the medication The nurse is assigned more clients than usual due to staffing issues The nurse has worked four 12-hour night shifts in a row The nurse has worked on the same unit for five years

The nurse works in the intensive care unit (ICU) The nurse was interrupted when preparing the medication The nurse is assigned more clients than usual due to staffing issues Rationale: There are a number of reasons for near misses and making medication errors, including heavy workload and inadequate staffing, distractions, interruptions and inexperience. Fatigue and sleep loss are also factors, especially for nurses working in units with high acuity clients such as the ICU.

The public health nurse is working at a screening clinic for sexually transmitted infections. A client has tested positive for human immunodeficiency virus (HIV). Which action should the nurse plan for next? The positive test result must be reported to the local health department. The client should be medicated with tenofovir/emtricitabine for pre-exposure prophylaxis. The client will be required to notify all past sexual partners. The client will need extensive teaching about acquired immunodeficiency syndrome (AIDS).

The positive test result must be reported to the local health department. Rationale: Many sexually transmitted infections (STI) are reportable at the state and federal levels, including HIV. A newly diagnosed HIV infection must be reported to the local health department, therefore the nurse should plan for that next. The client may be asked to provide information about recent sexual contacts who may have been exposed to the virus, but not necessarily all past sexual partners. Pre-exposure prophylaxis (PrEP) medications such as tenofovir/emtricitabine (Truvada) are intended for clients who are at high-risk of contracting HIV. They are not intended to treat an active HIV infection. AIDS is a complication that may develop in clients who are HIV positive. A client who is HIV positive does not necessarily have AIDS.

The home-health nurse is assessing the client's home for fire hazards. Which of the following findings requires intervention? An iron facing upright on the ironing board The use of the oven as storage space for food items The use of an ashtray on the porch to smoke outside Smoke alarms installed on every floor of the house

The use of the oven as storage space for food items It requires intervention if the client is using the oven as storage space for food items, as this can create a fire hazard if the oven is turned on accidentally. If the client continues to smoke, smoking should occur outside of the house and away from oxygen, and cigarettes should be disposed of in an ashtray. It is correct for smoke alarms to be installed on every floor of the house and for an iron to be placed in an upright position (never facing down) on an ironing board.

A nurse is inspecting medical equipment in a client's room. Which finding requires immediate intervention? The sharps container is halfway full. The bed is connected to the wall with a three-prong electric plug. The vital signs machine is unplugged. There is a water spill behind the infusion pump.

There is a water spill behind the infusion pump. Rationale: Electrical cords for equipment should be kept away from water sources. The nurse should ensure the water spill is cleaned to avoid the risk of electrocution. A sharps container that is halfway full does not require immediate attention. A three-prong plug is recommended to provide a ground for electrical equipment. An unplugged vital signs machine can drain the battery. However, reconnecting the machine is not the priority action.

The nurse is performing an initial home assessment for a client who is at risk for falls. Which of the following findings indicates the need for client education? Night light plug-ins present in each room Raised toilet seats in the bathrooms Shower chair present in the client's bathroom Throw rugs on the flooring surfaces

Throw rugs on the flooring surfaces The presence of throw rugs increases the risk for falls, and they should be removed. Night lights, raised toilet seats, and shower chairs are all appropriate devices to help reduce the risk of falling.

The nurse is performing a home visit for an older adult client with Alzheimer's disease. Which of the following observations should be a priority for the nurse to address? Throw rugs on the kitchen floor Good lighting in the stairwell Handrails in the bathtub Lamps plugged directly into wall outlets

Throw rugs on the kitchen floor Rationale: Safety, especially falls, is a concern for clients with Alzheimer's disease. The home should ensure good lighting, especially in dark areas such as stairwells. There should be handrails on the stairs and in the tub/shower. Showers should have nonskid mats. The client and caregivers should ensure there are no extension cords in use, as they can be a fire and trip hazard. Throw rugs are also a trip hazard and should be removed from the kitchen floor. This is the priority issue the nurse should address.

The nurse asks an unlicensed assistive person (UAP) to help with repositioning of a client in bed. Which actions by the nursing staff support correct ergonomics and safe client handling? Select all that apply. Use a friction-reducing device/sheet underneath the client. Lower the head of the bed into a flat position. Instruct the client to hold their breath. Coordinate lifting together by counting to three. Ask a visiting family member to help. Adjust the height of the bed to hip level.

Use a friction-reducing device/sheet underneath the client. Lower the head of the bed into a flat position. Coordinate lifting together by counting to three. Adjust the height of the bed to hip level.

A nurse is caring for a client who has left-sided weakness and is being transported to radiology by wheelchair. Which of the following actions should the nurse plan to take when transferring the client from bed? Raise the bed to the nurse's working height Use a gait belt when assisting the client Assist the client by lifting under the arms while they stand Place the chair on the client's right side

Use a gait belt when assisting the client For a bed-to-chair transfer, the bed should be in its lowest position to prevent client injury. Using a gait belt will help prevent injury to the client and nurse and allows the nurse to have a firm grip on the client as needed. During any patient-transferring task, if any caregiver is required to lift more than 35 lb of a client's weight, consider them to be fully dependent and use assistive devices for the transfer. The wheelchair should be on the client's strong side.

A nurse is providing injury prevention strategies to a client with hemiparesis. Which intervention will decrease the risk of environmental injury? Performing passive range of motion to the affected extremities Placing the wheelchair at the foot of the bed Using a sliding board to transfer from bed to chair Securing the affected arm in a sling while ambulating

Using a sliding board to transfer from bed to chair Rationale: Clients with hemiparesis are unable to move one side of their body. Using a sliding board to transfer from bed to chair decreases the risk of falls. Performing passive range of motion to the affected extremities promotes muscle tone but does not directly prevent injury. Placing a wheelchair at the foot of the bed increases the risk of falls. Wheelchairs should be placed close to the client on the unaffected side. Securing the affected arm in a sling decreases the weight of the extremity and prevents shoulder injuries.

The nurse is caring for a client diagnosed with gastroenteritis, caused by a Salmonella infection. Which intervention should the nurse implement to prevent transmission of this infection? Wash hands with soap and water after client contact. Isolate the client in a single room without a roommate. Place the client on contact precautions. Wear two pairs of gloves when changing linens.

Wash hands with soap and water after client contact. Rationale: Salmonella is a bacterium and one of the primary causes of foodborne illnesses such as gastroenteritis. Bacteria transmission usually occurs through ingestion of the organisms via contaminated foods and the oral-fecal route. The Centers for Disease Control and Prevention (CDC) recommends using standard precautions for this infection; therefore, the best way to prevent spread of the infection is to perform handwashing before and after client contact, using soap and water.

Parents call the emergency department to report that their toddler has swallowed drain cleaner. The triage nurse instructs them to call for emergency transport to the hospital and suggests that the parents give the toddler sips of which substance while waiting for an ambulance? Water Lemonade Tea Soda

Water Rationale: Ingestion of drain cleaner is corrosive to the esophagus and should not be vomited up. The client should be encouraged to take sips of water or milk if they are able to dilute the corrosive substance. Lemonade may worsen the corrosive effect. Soda and tea would be inappropriate to administer.

The nurse is caring for a client who is receiving prescribed cytotoxic chemotherapy. Which safety precautions should the nurse implement with this client? Wear gown, gloves, and face shield when handling body fluids for 3 to 5 days after the last dose Require visitors to wear gowns and gloves while in the room Adhere to universal precautions when changing IV tubing every 2 to 4 hours Dispose of contaminated needles, syringes, vials, and ampules in red biohazard containers

Wear gown, gloves, and face shield when handling body fluids for 3 to 5 days after the last dose Rationale: Blood and body fluids are contaminated with drugs or metabolites for about 3 to 5 days after a dose. Therefore, the nurse should wear appropriate PPE when handling patients' clothing, bed linens, or excreta. Gloves alone may be inadequate in protecting the nurse from exposure to cytotoxic agents. Visitors are not required to wear a gown and gloves when in the room. Dispose of contaminated materials (e.g., needles, syringes, ampules, vials, IV tubing, and bags) in puncture-proof containers labeled "Warning: Hazardous Material." Chemotherapy waste containers are typically yellow in color.

The nurse is caring for a client with a chest tube. The client is confused and keeps attempting to pull out the chest tube. The nurse applies soft restraints on both of the client's wrists. Is the nurse acting appropriately? Yes, the nurse should apply a restraint to protect the client from self-injury, and then must contact the HCP. No, the nurse should first medicate the client with a sedative until the client stops pulling on the chest tube. No, the nurse needs a written order from the HCP before the restraints can be applied. No, the nurse should first ask a family member to stay with the client at all times.

Yes, the nurse should apply a restraint to protect the client from self-injury, and then must contact the HCP. Rationale: Clients have the right to be free from physical or chemical restraints used for the purpose of discipline or staff convenience. A soft wrist restraint can be applied before a doctor's order is given, but the nurse must contact the HCP immediately after the restraint is applied to obtain the order. Sedatives are not appropriate for this client because they can make the client's confusion worse and cause central nervous system and respiratory depression. Asking a family member to stay with the client is not an appropriate intervention.

The nurse has attended a staff education conference about incident reporting. Which of the following statements by the nurse indicates a need for further teaching? "An incident report should be submitted for a client who fell after receiving an opioid analgesic." "Any type of medication error requires the submission of an incident report." "An incident report should be submitted when a client has an allergic reaction to a newly prescribed antibiotic." "Any time a prescription is written for the wrong client, an incident report should be submitted."

"An incident report should be submitted when a client has an allergic reaction to a newly prescribed antibiotic." Rationale: An incident report is an agency record of an accident or unusual occurrence in the healthcare setting, which can be used to help prevent future incidents or accidents. It is the correct understanding of incident reporting if the nurse states that a client fall, medication error, and an incorrect prescription that is written for a client require incident reports. These are all considered errors or adverse/preventable events and should be reported to identify strategies to reduce human error and adverse events in the future. Clients may have unknown allergies to certain medications, and this requires prompt treatment of the allergy depending on severity as well as documentation in the medical record of the allergy, however, this is not considered an actual accident that requires incident reporting.

The home health nurse is conducting a safety visit with the family of a toddler who is crawling. Which statement by the parent would require follow-up by the nurse? "We use a thermometer to test the temperature of the bath water." "A gate was installed in the entryway to the kitchen." "There are coverings placed over all electrical outlets." "Any furniture with sharp edges we cover with a blanket.

"Any furniture with sharp edges we cover with a blanket. Rationale: Checking the water temperature reduces the risk of burns. Toddlers are unsteady and can fall easily on unsecured rugs. Toddlers put everything in their mouths so small objectives are a choking hazard. Covering the electrical outlets will prevent them from placing items in the outlet.

The nurse is providing staff education on reducing hospital acquired infections by eliminating potential reservoirs of infection. Which of the following statements should be included in the teaching? "Client bedrails should be disinfected regularly using hospital approved wipes." "It is a good idea to wear gloves when touching door handles." "Documentation areas are cleaned less often than client rooms." "Soiled linens should remain in the client's laundry bin until discharge."

"Client bedrails should be disinfected regularly using hospital approved wipes." Rationale: Research has shown that one of the biggest reservoirs of infection is the client's bedrails. These must be thoroughly sterilized between patients. Gloves are a potential source of infection. If someone is wearing gloves and touches a reservoir then touches the door handle, the door handle becomes a reservoir. Clients, visitors, healthcare providers, and facility staff may touch door handles multiple times each day. Office supplies, computer mice, and keyboards are all potential sources of infection and need to be diligently cleaned. Linen bags should be brought directly to the soiled utility room.

The nurse is educating a client who has age related macular degeneration on home safety practices. Which of the following statements is appropriate? "Ask for written instructions for your medications." "Keep lights in your home dim to avoid glare." "Color-code the controls on your kitchen appliances." "Watch television with closed captions."

"Color-code the controls on your kitchen appliances." Rationale: Color-coding kitchen appliance controls helps to ensure that the client is able to operate the appliances safely. An example of this is stove burner knobs; turning on the incorrect burner increases the risk of burns and fire. Lights should be bright, and written materials such as medication education and closed captions may be difficult for the client to read and should be avoided.

The nurse has taught a client about measures to reduce the risk of repetitive stress injuries. Which of the following statements by the client indicates the need for further teaching? "Varying the types of sports that I play may reduce the risk of acquiring a repetitive stress injury." "I will limit the amount of time I use electrical tools each week to reduce repetitive vibration to my joints." "Frequent lifting of heavy objects will increase muscle strength and reduce my risk of repetitive stress injuries." "I will ensure that I have running shoes with proper padding in the soles to reduce impact on the joints in my legs."

"Frequent lifting of heavy objects will increase muscle strength and reduce my risk of repetitive stress injuries." Rationale: Repetitive stress injuries are caused by sports, occupations, and hobbies that cause repetitive motion, causing strain on the joints and tendons. Frequent lifting of heavy objects increases the risk of repetitive stress injuries and requires further teaching. Varying the types of sports that are played can reduce the risk of acquiring a stress injury due to using the same motion frequently when training often for one particular sport (i.e., tennis elbow). Clients that use electrical tools that vibrate either for hobbies or for work are at risk for injury and these tools should be limited, if possible. Proper padding in running shoes and allowing for rest each week will reduce the risk of running-related stress injuries.

The nurse is providing teaching to client on the use of range of motion exercises while on bedrest. Which statement made by a client indicates the need for further teaching? "I will move each joint through the full range at least 3 times." "I should raise my leg with my foot 6 inches off the bed." "I can move the joint until I feel resistance." "I will hold my leg when someone rotates the joint."

"I will hold my leg when someone rotates the joint." Rationale: Active range of motion exercises are movements that the client does independently. The client should be instructed to perform the active range of motion exercises. The nurse should instruct the client to perform the movement until resistance is felt. When exercising a joint, the movement should be done at least 3 times. When performing leg exercises, the client should lift the foot about 6 to 12 inches off the bed. Passive range of motion exercises are done with someone performing the exercise.

The occupational health nurse is teaching a client about measures to reduce the risk of carpal tunnel syndrome. Which of the following should the nurse include in the teaching? "Geometrically designed keyboards may assist with reducing strain on your fingers and wrists." "Lower your chair height so that your wrists are flexed." "Take frequent breaks from keyboards to perform other finger motions." "Stretch your fingers and wrists in the morning before work."

"Geometrically designed keyboards may assist with reducing strain on your fingers and wrists." Rationale: Carpal tunnel syndrome is the most common repetitive stress injury. Clients should be taught proper ergonomics to reduce the risk of developing carpal tunnel syndrome, including stretching wrists frequently during the day while at work and typing, adjusting the chair height so that elbows are at a 90-degree angle without flexion of the wrists. Clients should be taught to take frequent breaks from keyboards in addition to typing on keyboards found on cellphones and handheld devices. A geometrically designed keyboard may reduce strain on fingers and wrists as well as adjustable height desks to allow for alterations in positions throughout the day.

The nurse is reviewing the communicable disease policy about what information needs to be provided to the health department. Which statement by the nurse indicates the need for additional education about the policy? "The results of testing should be provided." "I will need to report the onset of symptoms." "HIPAA prevents the reporting of personal information." "Patient information such as name, age, and gender are reported."

"HIPAA prevents the reporting of personal information. Rationale: This type of required reporting uses personal identifiers and enables the states to identify cases where immediate disease control and prevention are needed. Each state has its own laws and regulations defining what diseases are reportable. The list of reportable diseases varies among states and over time. HIPPA does not apply to reportable diseases.

The nurse is assessing a client for the risk of allergic reactions to products commonly used in the healthcare setting. Which of the following statements by the client requires follow-up? "I have an air purifier in my home for allergies to pet dander." "I take a daily antihistamine due to dust-mite allergies." "I avoid eating bananas because they make me itchy." "I have a family history of anaphylaxis due to bee stings.

"I avoid eating bananas because they make me itchy." Rationale: Clients should be assessed for risk of allergic reactions to products commonly used in the healthcare setting, such as iodine, adhesive tape, and latex. A sign of allergic reaction includes itching after eating a product. Allergies to foods such as bananas, kiwis and avocados are known to also cause allergic reactions to latex in clients, therefore itching after ingesting bananas may indicate risk of allergic reaction to latex. Allergies to pet dander, dust mites and a family history of allergies to bee venom do not increase the client's risk of allergic reaction to common healthcare products.

The nurse is assessing a client for allergies prior to a scheduled magnetic resonance image (MRI) with contrast. Which of the following statements by the client requires follow-up? I take a daily antihistamine for nasal congestion due to pet dander." "I get itchy when I put iodized salt on my food." "I had a reaction in the past when I received blood products." "I have an air purifier in my bedroom for dust-mite allergies."

"I get itchy when I put iodized salt on my food." Rationale: Clients who are undergoing diagnostic testing with contrast medium should be assessed for allergies to iodine-containing food such as shellfish, cabbage, kale, and iodized salt, which could cause an adverse reaction. Allergies to pet dander, dust mites, and a previous blood transfusion reaction do not place the client at an increased risk of an adverse reaction to the contrast medium.

The nurse is interviewing an older adult client in the outpatient clinic about home safety. Which statement by the client indicates the need for intervention? "My toilet has a raised seat on it." "I like to keep the lighting low because it hurts my eyes." "I wear non-slip socks when I am walking around the house." "My son installed a handrail next to my toilet."

"I like to keep the lighting low because it hurts my eyes." Rationale: Low lighting increases the risk of falling, therefore the nurse should educate the client on the need to increase light brightness or add lighting to the home. A raised toilet seat, non-slip socks, and handrails are all appropriate safety devices.

The nurse is teaching a client newly diagnosed with HIV about virus transmission. Which statement by the client would indicate to the nurse the need for further teaching? "I should avoid cooking for my family." "I will notify all of my sexual partners." "I am going to wear a medical alert bracelet." "I can still use the community gym."

"I should avoid cooking for my family." Rationale: HIV is transmitted through direct contact with bodily fluids, such as unprotected sex or sharing of used needles. HIV is not transmitted through casual contact, such as preparing food or using shared gym equipment. Clients who are newly diagnosed with HIV should be instructed to contact previous sexual partners.

The nurse is caring for a client who uses oxygen at home. Which of the following statements by the client warrants additional assessment by the nurse? "I keep my spare tanks in the bedroom closet." "I use a beeswax- based moisturizer when my nose gets dry." "I keep the oxygen concentrator outside of the bathroom when I shower." "I wear my oxygen when I'm preparing our dinner."

"I wear my oxygen when I'm preparing our dinner." Rationale: Clients should always maintain at least two meters (approx. 6 feet) between a fire and a portable oxygen concentrator and accessories. This includes gas stovetops. Clients may need to wear oxygen when showering or bathing, but it is important the concentrator itself does not get wet. Extension tubing can be used to facilitate this. Clients will have spare tanks at home in case of power failure. They should be stored in a secure location away from open flame and heat.

The nurse is teaching a group of parents about measures to reduce the incidence of unintentional poisoning in toddlers. Which of the following statements by a parent indicates a need for further teaching? "I will carefully evaluate the products my child's toys are made of to reduce the likelihood that they contain lead." "I will keep houseplants on the ground to ensure they do not get knocked over and cause injury." "I will never leave my child unsupervised around standing water sources." "All household chemicals will be kept on a top shelf, out of reach from my child."

"I will keep houseplants on the ground to ensure they do not get knocked over and cause injury." Rationale: Major causes of unintentional injury in toddlers are accidental poisoning due to ingesting toxic substances, such as plants, chemicals, and medications. Additional causes include exposure to lead in toys, paint, and ceramic dishes as well as drowning. The nurse should follow up if the parent states that houseplants should be kept on the ground. Toddlers are curious and often stick objects in their mouths, including plants, which can sometimes be poisonous if ingested. Plants should be kept out of reach of toddlers.

The nurse is teaching a client about protective measures to reduce the risk of electrical shock while using medical devices. Which of the following statements by the client indicates the need for further teaching? "I will repair any frayed or damaged equipment cords with electrical tape." "I will place protective covers over wall outlets around my younger children." "I will use only grounded outlets and plugs to power my medical equipment." "I will avoid overloading outlets with too many appliances at a time."

"I will repair any frayed or damaged equipment cords with electrical tape." Rationale: It indicates the need for further teaching if the client states they will repair frayed or damaged electrical cords with electrical tape. Devices that indicate damage should not be used and should be evaluated and repaired by an electrician. It indicates a correct understanding of the use of protective equipment to reduce electrical shock by placing protective covers on electrical outlets around children, to use only grounded outlets, and to avoid overloading outlets with too many appliances.

The nurse has provided strategies to a client about infection prevention. Which of the following statements by the client indicates the need for further teaching? "I will use antimicrobial soap and hot water to wash my hands." "I will wash all raw fruits and vegetables before eating." "My fingernails should be cut short and clean." "Sharing my toothbrush is not a good practice."

"I will use antimicrobial soap and hot water to wash my hands." Rationale: Antimicrobial soap is not needed for routine handwashing and can lead to resistant organisms. Raw fruits and vegetables can carry infectious organisms and should be washed before eating. Fingernails should be cut and clean to prevent harboring of organisms. The nurse should instruct the client to avoid sharing personal grooming items, such as toothbrushes.

The nurse is instructing a group of unlicensed assistive personal in the correct use of personal protective equipment. Which statement by the UAP indicates understanding of the correct protocol? "I only need to wear a mask during influenza season." "I will wear gloves when performing hygiene care." "I should wear a personal protective gown when assisting with meals." "I have to put on the protective equipment when entering the room."

"I will wear gloves when performing hygiene care." Rationale: Gloves should be worn for all contact with blood and body fluids, nonintact skin, and mucous membranes for handling soiled items during hygiene care. Masks are worn when exposed to airborne and droplet respiratory disorders, such as tuberculosis or Covid-19, and can be worn any time of year. Gowns should be worn during procedures that are likely to cause splashes of blood or body fluids. Protective equipment is put on before entering a room.

A nurse is providing education to a client with diabetes type 2 on using a blood glucose monitor at home. Which client statement indicates an understanding of the teaching? "I will keep the used lancets on the counter until I can dispose of them properly." "I will do the quality control test every time I use the glucose monitor." "I will place the test strips in a plastic bag when I travel." "I will write the date I opened the test strips on the container.

"I will write the date I opened the test strips on the container. Rationale: Expired test strips can produce inaccurate results. Writing the date on the bottle prevents test strips from being used beyond their expiration date. Quality control tests do not need to be performed every time the glucose monitor is used. The nurse should instruct the client to perform the quality control test as recommended by the device manufacturer. Test strips should be kept in their original container. Light and moisture can affect their functionality. Lancets should be disposed of immediately after use. Exposed lancets can cause a needlestick injury.

The nurse is teaching a client about fire safety in the home. Which of the following statements by the client indicates the need for further teaching? "I will change the batteries in smoke alarms on special days, such as the first of the year." "I will keep my fire extinguishers in locations that are most prone to fires, such as the kitchen." "In the event of a fire, I will open all the windows in my house." "In the event of a fire, I will move through smoke-filled areas with my head as close to the floor as possible.

"In the event of a fire, I will open all the windows in my house." Rationale: It requires further teaching if the client states that they will open all windows and doors during a fire. The client should be taught to contain the fire, if possible, by closing windows and doors, which deprives the fire of oxygen. It is correct for the client to change smoke alarm batteries annually and doing so routinely on a special day helps clients to remember to do so. Fire extinguishers should be kept in locations that are prone to fires, such as the kitchen or near grills. In the event of a fire, clients should move with their heads as close to the ground as possible to prevent smoke inhalation.

The nurse is reviewing the plan of care with a client who has a prescription to remain supine for 24 hours following a procedure. Which statement should the nurse make to the client regarding positioning? "Keep your knees flexed." "Place a pillow under your legs." "Rotate your hips side to side to relieve pressure." "Raise your arms over your head to stretch."

"Place a pillow under your legs." Rationale: When caring for a client with a prescription for the supine position, the nurse should implement interventions to prevent pressure injuries. The supine position increases pressure on the client's heels and coccyx. To alleviate this pressure, the nurse should instruct the client to keep a pillow under their legs, this will float the heels off the bed and prevent pressure. Instructing the client to keep knees flexed will increase pressure on heels. Raising arms above the head does not reduce pressure. Rotating hips can increase pressure.

The nurse is teaching a client who is newly diagnosed with diabetes type I about needle disposal. Which statement should the nurse include in the teaching? "Sharps with safety mechanisms can be discarded directly into regular trash." "Put used sharps in a strong, plastic container that has a lid." "Plastic containers that are full of sharps should not be placed in the regular trash." "Sealed containers must be dropped off at a local acute care facility."

"Put used sharps in a strong, plastic container that has a lid." Rationale: Clients should be taught to put used sharps in a strong, plastic container. When the container is 3/4 full, put the lid on, seal it with tape, and label "do not recycle." It is then acceptable to put the container in the household trash or bring it to a drop-off location such as a hospital, pharmacy, health department, or police or fire station. Sharps that retract after use should be disposed of like all other sharps.

The charge nurse observes a staff nurse carrying soiled linen in the hallway from a client who is receiving chemotherapy. Which statement by the charge nurse would be most appropriate? "Soiled linens should be held away from the body and placed in a linen cart or bag before leaving the room." "Soiled linens should be left in the client's room to prevent the spread of biohazardous and infectious materials." "Linens should be changed weekly to prevent exposure to cytotoxic agents." "Linens should be held closely to reduce the risk of any biohazardous or infectious materials from becoming airborne."

"Soiled linens should be held away from the body and placed in a linen cart or bag before leaving the room." Rationale: Blood and body fluids are contaminated with cytotoxic drugs or metabolites for about 3 to 5 days after a dose. Therefore, the nurse should wear appropriate PPE when handling patients' clothing, bed linens, or excreta. Linens should be placed in a specially labeled linen cart or plastic bag before being taken to the soiled utility room. It is important that meticulous hygiene is administered to the patient undergoing chemotherapy to help prevent infection. Therefore, linens should be changed as needed. Linens should be held away from the body to prevent contamination of the nurse's clothing.

The nurse is providing teaching about car seats to the parents of a 30-pound child. Which of the following should be included in the teaching? "The buckled chest clip should be 1 inch below the armpit level of the child." "A child can safely ride in the front seat while in a booster seat." "The child can be placed in a forward-facing car seat with a harness." "The car seat can move side-to-side 1-2 inches after being secured."

"The child can be placed in a forward-facing car seat with a harness." Rationale: A child of 30 pounds can sit in a forward-facing car seat or booster. The buckled clip needs to be at the armpit not below. Children cannot ride it the front seat, regardless of weight. All seats should be securely fastened and not be able to move at all.

The nurse has attended a staff education conference about assessing for factors that impact a client's ability to ambulate. Which of the following statements by the nurse indicates a need for further teaching? "The length of time a client has been in bed may impact the ability to ambulate without assistance." "The client should be medicated right before they attempt to ambulate." "Clients should be assessed for the ability to understand directions prior to attempting ambulation." "Client's range-of-motion ability should be assessed prior to attempting ambulation."

"The client should be medicated right before they attempt to ambulate." Rationale: Several factors influence a client's ability to balance and ambulate safely. It requires further teaching if the nurse states that narcotics will improve balance and safety. While it is important to address pain levels and ensure comfort prior to ambulation, several medications, such as narcotics, sedatives, and tranquilizers may cause drowsiness, dizziness, weakness, and orthostatic hypotension and can hinder the client's ability to walk safely. The length of time a client is in bed can cause weakness and impair ability to transfer and ambulate. Clients should be assessed for the ability to understand directions during ambulation with or without assistive devices to ensure safe transfers. Limitations in range-of-motion or lower extremity strength may hinder ability to balance and transfer and may require assistive devices as needed.

The nurse is teaching a client who just had a leg cast removed about ways to strength the calf muscle to reduce the stress due to inactivity. Which of the following statements by a client indicates a need for further teaching? "I will need to do the exercise 2 to 3 times a week to build strength." "I should do 8 to 12 repetitions of each exercise." "I will need to do several sets for each exercise daily." "The faster I move through the exercises, the sooner my muscle strength will return."

"The faster I move through the exercises, the sooner my muscle strength will return." Rationale: Isometric exercises are used for strengthening muscles without moving the joint. These can be used for maintaining strength in immobilized muscles in casts or traction. It is recommended that clients should do the exercises consistently two or three times a week to build strength. The nurse should instruct the client to perform the eight to 12 repetitions of each exercise for one to three sets. Clients should move slowly through each exercise to help build muscle strength. Moving too quickly can increase stress and damage the muscle.

The nurse is teaching a group of clients at the community center about burn prevention for children. Which of the following should be included in the teaching? "The handles of the pots on the stove must be turned inward." "The hot water tank should be set at 150 degrees." "Infants can be bathed in the kitchen sink." "Electrical outlets do not need to be covered until the infant is crawling."

"The handles of the pots on the stove must be turned inward." Rationale: Turning the handles in will limit the risk of the child reaching up and pulling a hot pot onto themselves. Hot water tanks should be set at 120 degrees or less to decrease likelihood of scalding. Bathing in the sink increases the risk that the infant will turn the hot water on. Electrical outlets should be covered to prevent anything from being inserted into it.

The nurse is providing room orientation to a client who is newly admitted. When instructing the client how to alert the staff, which statement should the nurse make? "You can call the client care station with your phone." "Let us know when you need something during hourly rounding." "Use the call light device when you need assistance." "You can call the hospital operator to have the staff paged."

"Use the call light device when you need assistance." Rationale: The best communication tool for clients to use is the call light. The hospital operator directs calls to the correct unit or provides announcements regarding emergencies. Hourly rounding is when staff visually observes clients and provides basic care, such as bathroom visits. Clients should still alert staff if needed and not wait for the rounding schedule. Calling the client care station may delay in alerting the staff.

A nurse is assisting a healthcare provider with a sterile procedure and prepares to pour a solution onto a piece of sterile gauze. In what order should the nurse perform the following steps when pouring sterile solution? Place the steps in order, and all steps are to be used. 1. Pour 1 to 2 mL into a receptacle. 2. Place the bottle cap face-up on a clean surface. 3. Pour the solution onto the gauze. 4. Perform hand hygiene. 5. Pick up the bottle with the label facing toward the palm. 6. Remove the bottle cap.

4, 6, 2, 5, 1, 3 4. Perform hand hygiene. 6. Remove the bottle cap. 2. Place the bottle cap face-up on a clean surface. 5. Pick up the bottle with the label facing toward the palm. 1. Pour 1 to 2 mL into a receptacle. 3. Pour the solution onto the gauze.

The charge nurse is required to recommend a client that can be discharged in the next hour due to a disaster plan activation. The nurse should recommend which client for discharge? A client post-laparoscopic cholecystectomy with a prescription for a soft diet A client with a comminuted pelvic fracture who is taking oral analgesics A client with atelectasis on oxygen via nasal cannula A client with a foot ulcer who is receiving intravenous antibiotics

A client post-laparoscopic cholecystectomy with a prescription for a soft diet Rationale: A postoperative client who is tolerating oral intake is considered stable for discharge. A laparoscopic cholecystectomy is minimally invasive, and clients are usually discharged within a day. A client with a comminuted pelvic fracture cannot be mobilized until treated. A client with atelectasis who requires oxygen therapy is not stable for discharge within an hour. A client receiving intravenous antibiotics is not ready for discharge. Intravenous therapy requires care management collaboration prior to discharge.

An emergency department (ED) charge nurse is receiving training on providing assignments to unit nurses who are floated to the ED during a disaster activation. Which assignment is appropriate for a nurse from an orthopedic unit? A client who had a myocardial infarction with continuous cardiac monitoring A client with end stage renal disease who is receiving hemodialysis A client with a femur fracture who is placed in skin traction A client who had acute respiratory failure with mechanical ventilation

A client with a femur fracture who is placed in skin traction Rationale: The charge nurse should assign a client that closely matches the nurses' competencies. A client on skin traction is an appropriate client to assign to an orthopedic nurse. Skin traction is applied to clients who have fractures. A client who requires cardiac monitoring is not an appropriate client to assign to an orthopedic nurse. Cardiac monitoring requires telemetry training. Taking care of a client on hemodialysis requires specialized training. Providing care to a client on mechanical ventilation is an advanced competency for critical care nurses.

During a disaster activation, a nurse is tasked with recommending the most stable client for discharge within the hour. Which client should the nurse recommend? A client with newly diagnosed diabetes type II who requires medication teaching A client post-bowel resection who has a new colostomy appliance A client with an ankle fracture who uses crutches for ambulation A client with a sacral wound who is receiving negative pressure wound therapy

A client with an ankle fracture who uses crutches for ambulation Rationale: A client with an ankle fracture who requires crutches is the most stable client for discharge. The client can ambulate with an assistive device. A client who needs medication assistance for a newly diagnosed disease is not appropriate for discharge. The client requires a case management consult. A client who has had major surgery is not the most appropriate client for discharge. A new colostomy requires extensive client education. A client on negative pressure wound therapy (NPWT) is not an appropriate client for discharge within an hour. Arrangements for home use of NPWT is a lengthy approval process.

The nurse is documenting an occurrence in which the nurse identified that the wrong site was marked by the surgeon prior to the procedure being performed. Which type of practice error should the nurse document as having occurred? An adverse event A near-miss event A sentinel event A root cause analysis

A near-miss event Rationale: A near-miss event is the identification of any event or situation that might have resulted in client harm, but the harm did not occur due to timely intervention by healthcare staff. The nurse should document the identification of the wrong surgical site marking prior to the actual surgery as a near-miss. Near-misses should be documented as they occur so that root cause analyses can be performed to identify human factors that could be avoided to prevent a similar situation from occurring. An adverse event and a sentinel event occur when actual harm or death is inflicted on a client due to healthcare errors.

The nurse is documenting an occurrence in which the incorrect mole was removed from a client's leg. Which type of practice error should the nurse document as having occurred? A sentinel event A near-miss event A never-event An unpreventable event

A never-event Rationale: The nurse should document the incorrect removal of a client's mole as a never-event. Never-events are defined by the Joint Commission as surgeries on the wrong body part, foreign objects placed in the client after surgery, and mismatched blood transfusions. A sentinel event is one in which serious injury or death occurred due to errors. A near-miss event is the identification of any event or situation that might have resulted in client harm, but the harm did not occur due to timely intervention by healthcare staff. An unpreventable event is one in which death or client injury would occur in the absence of a medical error.

A charge nurse is assigning a room to a client with a history of moderate Alzheimer's. The charge nurse will assign the client to a room in which area of the unit? Next to the client activity room At the end of the hallway In front of the elevator Across from the medication room

Across from the medication room Rationale: Clients with moderate Alzheimer's may have personality and behavioral changes that lead them to wander and get lost. Medication rooms are frequently used by nurses. This placement ensures frequent visual checks of the client. Assigning the client to a room at the end of the hallway is not appropriate. Most stairwells are at the end of hallways and can be an area for the client to escape. A room in front of the elevator is not appropriate for a client with Alzheimer's. Assigning the client next to an activity room provides overstimulation. Noise should be kept to a minimum.

The charge nurse is observing a newly hired nurse use a fire extinguisher for a small fire in a client's room. Which action by the newly hired nurse would require intervention by the charge nurse? Aiming the hose at the top of the fire Pulling out the fire extinguisher's safety pin Squeezing the handle to discharge material onto the fire Sweeping the hose from side to side until the fire is extinguished

Aiming the hose at the top of the fire Rationale: Correct technique for use of a fire extinguisher includes pulling out the safety pin, aiming the hose at the base of the fire, squeezing the handle to discharge the material, and sweeping the hose from side to side. It requires intervention if the nurse is observed aiming the hose at the top of the fire instead of the base.

A nurse is providing care to a client experiencing substance withdrawal. Which prescription by the healthcare provider requires clarification? Implement seizure precautions. Keep client NPO. Initiate cardiac monitoring. Ambulate ad lib.

Ambulate ad lib. Clinical manifestations of substance withdrawal include tremors, irritability, and unsteady gait. These manifestations increase the risk for falls. A prescription for ambulation as desired requires clarification. Substance withdrawal is one of the main risk factors for seizure activity. Seizure precautions are indicated. A nothing by mouth (NPO) order is indicated for a client experiencing substance withdrawal. Nausea and vomiting are common manifestations. Substance withdrawal can cause tachycardia and elevated blood pressure. Cardiac monitoring is indicated.

The nurse is participating in the implementation of a hospital's disaster response plan. Which of the following indicates correct understanding of disaster planning? All hospital staff must receive training on identifying signs of bioterrorism activities within the community. All hospital staff must receive training on handling of hazardous materials and decontamination. Annual drills are required and should include community-wide resources with a simulation of a large influx of clients. The hospital pharmacy is required to stockpile antibiotics and nerve agent antidotes in the event of a bioterrorist attack.

Annual drills are required and should include community-wide resources with a simulation of a large influx of clients. Rationale: All facilities are required to carry out internal and external disaster drills, one of which includes implementing community-wide resources and simulation of a large influx of clients in the event of a disaster. Typically, nurses, emergency department physicians and other medical providers are required to receive training on handling hazardous materials, decontamination and recognizing patterns of illness that indicate potential bioterrorism in the community. While it is ideal for pharmacies to stockpile antidotes to nerve agents and antibiotics, this is not a federal requirement, although resources are becoming more available for facility pharmacies to obtain these medications.

The nurse is conducting a compliance survey to identify adherence to infection control practices. While observing handwashing, which of the following findings would require intervention? Artificial nails are scrubbed using a brush for 30 seconds. Handwashing occurs for at least 15 seconds. Wrists are included in the washing. Soap is rinsed off from the wrist down to the fingertips.

Artificial nails are scrubbed using a brush for 30 seconds. Rationale: A nurse with artificial nails may harbor a large number and variety of microbes under the nails. The CDC and WHO both have recommendations regarding the avoidance of artificial nails in healthcare settings. Handwashing should occur for at least 15 seconds, include the wrists, and rinse water proximal to distal.

A nurse is preparing to obtain a blood specimen from a client. Which action should the nurse take? Labels the tubes before the specimen is obtained Writes the client's information directly on the tube Places a client label on the biohazard bag prior to transport Asks the client to state their name and date of birth

Asks the client to state their name and date of birth Rationale: The nurse should identify the client by asking them to state their name and date of birth. Ensuring client identification prevents the nurse from mistakenly obtaining blood from the incorrect client. The tube should not be labeled until after the blood specimen is collected. Writing the information directly on the tube can smudge and does not ensure proper identification of the specimen. Placing a client label on the biohazard bag does not ensure proper identification if the specimen is separated from the bag upon analysis.

The nurse is making client care assignments during a facility disaster drill. Which action by the nurse indicates correct understanding of assignments during a disaster? Assigning a nursing administrator to care for clients in the post-anesthesia care unit Assigning a nurse on the medical-surgical floor to perform triage on disaster victims in the emergency department Assigning a critical care nurse to care for emergent clients in the emergency department Assigning unlicensed assistive personnel to monitor a group of clients on a medical-surgical floor

Assigning a critical care nurse to care for emergent clients in the emergency department Rationale: Mass casualty events often require nurses to practice outside of their normal daily duties. Nursing administrators may be assigned to client care on units; however, they should be assigned to stable, predictable clients. Clients who are immediately post-operative in the post-anesthesia unit are in critical condition and should not be assigned to the nursing administrator. Nurses on the medical-surgical floor may be assigned to care for stable clients in the emergency department so that emergency room nurses can perform triage on critical patients. Critical care nurses may be reassigned to care for critical patients in the emergency department. Unlicensed assistive personnel cannot perform regular nursing assessments or interventions even in a disaster situation and should not be assigned to monitor or manage a group of medical-surgical clients.

The charge nurse is observing a newly hired nurse assess the client's ability to ambulate while transferring from a supine position in bed. Which of the following actions by the newly hired nurse requires intervention? Assisting the client to sit upright with legs dependent on side of bed for 10 seconds Assessing for upper and lower extremity weakness prior to standing by the side of the bed Assisting the client to stand by the side of the bed for at least 1 minute Assessing the client for dizziness or a sudden increase in heart rate prior to ambulation

Assisting the client to sit upright with legs dependent on side of bed for 10 seconds Rationale: When assisting clients to transfer from a supine position to ambulate, clients should be moved slowly and closely monitored for orthostatic hypotension and any weakness. It requires intervention if the nurse has the client sit and dangle the legs for 10 seconds. Clients should sit at the side of the bed for at least 1 minute for assessment of possible dizziness or vital sign changes. It is correct for the nurse to assess for extremity weakness prior to attempting to stand, assisting the client to stand for at least 1 minute prior to ambulation, and to assess the client for dizziness.

A dayshift nurse administered insulin aspart to a client at 0730. The nightshift nurse calls stating they forgot to document the administration of the same medication to the client at 0630. Which action does the nurse perform next? Provide the client with a cup of juice. Fill out an incident report. Check the client's capillary blood glucose. Notify the healthcare provider.

Check the client's capillary blood glucose. Rationale: The nurse's priority action is to assess the client for hypoglycemia due to the administration of a double dose of insulin. Insulin aspart is a rapid-acting insulin that can decrease glucose levels. Providing the client with a cup of juice is an intervention for correcting hypoglycemia. The nurse must first assess the client's glucose level. Filling out an incident report is important for quality improvement. However, it is not the priority action. Notifying the healthcare provider is important. However, the nurse must first assess the client's condition.

A nurse is preparing to assess a newly admitted client. Which action should the nurse take first? Explains the purpose of the assessment. Checks the client's wristband. Obtains a health history. Places the client in a supine position.

Checks the client's wristband. Rationale: The nurse should ensure proper identification of the client before providing care. The client's wristband will confirm name, date of birth, and medical record number. Explaining the purpose of the assessment is important. However, the nurse should ensure care is being provided to the correct client. Obtaining a health history should precede a physical assessment. However, the nurse must ensure the health history is being obtained from the correct client. Placing the client in a supine position is not a priority action. The nurse must first identify the client.

The nurse educator is reviewing the policy about needlestick injuries with a group of staff nurses. Which action should the nurse take first following a needlestick from a contaminated needle? Put the needle in a biohazard bag for testing Report to the Emergency Department Clean the site with soap and water Make an appointment to see the healthcare provider

Clean the site with soap and water Rationale: The puncture site and skin should be washed thoroughly with soap and water. Then the nurse should follow the next steps in the facility-specific protocol for when a needlestick occurs. Once the nurse has reported the incident to the supervisor, the nurse will be directed to seek immediate treatment.

The home health nurse is assessing a client's home for safety hazards. Which finding observed by the nurse would increase the client's risk for carbon monoxide poisoning? Client uses a wood-burning stove to heat the home. Client keeps their electric car stored in the garage. Client keeps a charcoal grill for cooking on the back patio. Client has a gasoline powered lawnmower stored in an outdoor shed.

Client uses a wood-burning stove to heat the home. Rationale: Carbon monoxide is an odorless, colorless gas that is toxic, and prolonged exposure can lead to brain damage or death. Risk factors for carbon monoxide poisoning include using gasoline-powered vehicles, lawnmowers, barbeques, and wood-burning units or stoves inside the home; therefore, a wood-burning stove places the client at an increased risk. The risk for carbon monoxide poisoning is reduced if these units are not in use in enclosed spaces.

The nurse is planning care for a client with methicillin-resistant staphylococcus aureus pneumonia. Which type of precaution should the nurse implement for this client? Airborne precautions Droplet precautions Contact precautions Standard precautions

Contact precautions Rationale: Contact precautions involve the use of barrier protection (e.g. gloves, mask, gown, or protective eyewear as appropriate) whenever direct contact with any of the body fluids is expected. When determining the type of isolation to use, one must consider the mode of transmission. The hands of personnel continue to be the principal mode of transmission for methicillin-resistant staphylococcus aureus (MRSA). Because the organism is limited to the sputum in this example, precautions are taken if contact with the patient's sputum is expected. Standard precautions are not enough to prevent transmission. Droplet and airborne precautions are not needed since the bacteria is not floating in the air.

The infection control nurse is evaluating a staff member putting on personal protective equipment (PPE) before entering the room of a client who is on droplet isolation. Which item should the staff member put on first? Gown Mask Gloves Face shield

Gown Rationale: When putting on PPE, the first item to put on is the gown, followed by the mask, face shield, and then gloves.

The nurse is caring for a client who is receiving the first dose of a newly prescribed intravenous antibiotic. Which finding observed by the nurse would indicate an allergic reaction? Development of oral thrush A decrease in heart rate from 80 to 72 Development of swelling in the lips An increase in systolic blood pressure from 102 to 115

Development of swelling of the lips Rationale: A sign of allergic reaction is swelling of the lips, mouth, and tongue. Other symptoms include the development of tachycardia and hypotension. Oral thrush, or candidiasis, is a yeast infection that can develop when a client is taking antibiotics.

A nurse is providing care to a client with chronic constipation. The client is on neutropenic precautions. Which prescription should the nurse clarify? High-fiber diet Abdominal x-ray Digital disimpaction Ambulate ad lib

Digital disimpaction Rationale: The nurse should clarify the prescription for digital removal of feces. Digital disimpaction can damage the mucosa along the rectal wall and increase the risk of infection. A client with neutropenia is immunosuppressed. A high-fiber diet is recommended for clients with chronic constipation and does not affect a client with neutropenia. An abdominal x-ray is not invasive and can help visualize blockage of the intestines. Ambulation increases peristalsis and is encouraged for clients with constipation. Ambulation is not contraindicated for a client with neutropenia.

The nurse is performing a dressing change for a client who has an abdominal wound healing by secondary intention. There is a moderate amount of sanguineous drainage on the old dressing. How should the nurse dispose of this dressing? Place in the client's regular trash bin Discard in a red biohazard waste bin Flush the dressing down the client's toilet Call environmental services to remove the waste

Discard in a red biohazard waste bin Rationale: A red bag marked BIOHAZARD is used to dispose of trash that contains liquid or semiliquid blood or other potentially infective material (OPIM), trash contaminated with blood or OPIM that would release these substances if compressed, and trash that is caked with dried blood or OPIM and is capable of releasing these materials during handling. Therefore, it is inappropriate to place the dressing in the client's trash bin. Dressings are not flushed down the toilet. It is unnecessary to call environmental services for this task.

A nurse is implementing seizure precautions for a client with tonic-clonic seizures. Which action should the nurse take? Raise all side rails on the bed Your Answer Instruct client to ambulate slowly to the restroom Ensure patency of the saline lock Position the overbed table in front of the client

Ensure patency of the saline lock Rationale: Clients with tonic-clonic seizures are at risk for injury due to rhythmic jerking of the extremities and changes in breathing patterns. The nurse should ensure intravenous access is patent in case pharmacologic treatment is required. Raising all of the siderails is considered a restraint. Clients should be instructed to call for assistance with ambulation. The client may fall and injure themselves during a seizure. Positioning the overbed table in front of the client can cause injury if the client experiences a seizure.

The nurse is performing client care in response to an external disaster in the community. Which of the following actions should the nurse take first? Evaluate clients for airway patency and effectiveness of breathing Assess clients for uncontrolled bleeding and apply pressure as indicated Evaluate clients for disability and immobilize the cervical spine as indicated Assess for exposure to hazardous materials and observe the client for injury

Evaluate clients for airway patency and effectiveness of breathing Rationale: The primary survey during a disaster response is performed to detect life-threatening injuries, which consists of checking for airway-breathing-circulation, disability, exposure, and then facilitation of family. The nurse should first assess clients for airway patency and quality of breathing, followed by the other actions.

The nurse is providing an in-service to the nursing staff on needlestick prevention using needle-based safety systems. Which of the following actions should the nurse demonstrate when placing the syringe with a needle in the sharps container? Recap the needle using the one-handed method Immediately engage the safety shield Leave the needle uncapped Remove the used needle using a needle clipper

Immediately engage the safety shield Rationale: Needle based safety systems have a one-handed safety mechanism built into the device. They should be engaged immediately after use. If the needle does not have a safety device, the safest way to dispose of a used needle is to immediately place it in a sharps disposal container. If a sharps disposal container isn't immediately available, recap the needle. Do not bend or break the needle and never remove a hypodermic needle from the syringe by hand. Recapping should be performed using the one-handed technique. Needle clippers make syringes unusable by clipping off the needle and are not used by nurses.

The nurse is caring for a client with a history of falls. The nurse observes the client attempting to get out of bed unassisted. Which safety device should the nurse implement for this client? Apply soft wrist restraints Raise all bed side rails Initiate a pressure bed alarm Set up an enclosure tent

Initiate a pressure bed alarm Rationale: When planning care for a client with a history of falls who is observed attempting to get out of bed unassisted, the nurse should implement safety devices that prevent injury. Restraints, such as wrist restraints, enclosure tents, and side rails, are physical devices that can limit the client's movement. The nurse should attempt less restrictive safety devices, such as pressure bed alarms. A pressure bed alarm is a safety device that alerts staff if the client attempts to get out of bed.

The nurse is planning care for a group of assigned clients. Which action by the nurse would require the use of sterile gloves? Placement of a peripheral IV Insertion of an indwelling urinary catheter Administration of prescribed rectal medication Performing perineal hygiene care

Insertion of an indwelling urinary catheter Rationale: Surgical asepsis is used for procedures that require sterile technique. Sterile technique is required for procedures that could introduce infectious agents to the client, such as placement of an indwelling urinary catheter, wound dressing change, or preparing injectable medications. Medical asepsis or clean technique is used for procedures such as placement of a peripheral IV, administering a rectal medication, or performing hygiene care.

The nurse is caring for a client who has an allergy to shellfish. Which of the following products should the nurse avoid when caring for this client? Iodine Chlorhexidine Alcohol Lidocaine

Iodine Rationale: Clients with shellfish allergies may have a cross-reaction with products that contain iodine, including Betadine. These products should be avoided to prevent an allergic reaction in the client. Lidocaine, alcohol, and chlorhexidine are not known to cause allergic reactions in clients with shellfish allergies.

The nurse is planning care for a client who is receiving a hematopoietic stem cell transplant. Which actions should the nurse take? Schedule open visiting hours with client's family Monitor the client's vital signs once a shift Provide the client with a pitcher of crushed ice Keep client care equipment in the room

Keep client care equipment in the room Rationale: Clients who are receiving stem cell transplants are at high risk for infection. The nurse should plan interventions that prevent infection. The nurse should keep all client care equipment in the room and use dedicated equipment to prevent the spread of infection. The nurse will limit the visitors to the client to prevent exposure. The nurse should provide fresh water, replacing it every hour. The nurse should monitor the client's vital signs every four hours.

A nurse is reviewing new prescriptions for a client with suspected meningitis. The client has a history of atrial fibrillation and is taking warfarin for disease management. Which prescription should the nurse clarify? CT scan of the head Blood cultures Lumbar puncture MRI of the spine

Lumbar puncture Rationale: The nurse should clarify the lumbar puncture order. Clients who are on anticoagulant medications are at risk of bleeding. A lumbar puncture may cause bleeding that can compress the spinal cord. Obtaining a blood culture is not contraindicated in a client taking anticoagulant medications. The nurse should apply direct pressure after the venipuncture. A computerized tomography (CT) scan of the head and a magnetic resonance imagining (MRI) of the spine are not invasive and will not affect the client taking anticoagulants.

The emergency department nurse manager is conducting a risk assessment for security planning. Which finding should the nurse manager identify as a potential risk for workplace violence? Nursing is understaffed on the nightshift Card-activated door locks are present Security staff are hired and trained by the organization Emergency department wait times are less than an hour

Nursing is understaffed on the nightshift Rationale: Risk factors for workplace violence include, but are not limited to, working when understaffed, working at night, high worker turnover, inadequate security personnel, and long waits for clients. Research indicates the importance of security staff who are specifically hired by the organization (and not by an outside organization) and trained to the unique issues related to the grounds of the organization. Card-activated door locks are a method of providing increased security and are not a weakness.

The nurse is caring for a client with lower extremity paralysis who uses a wheelchair and is non-weight bearing. Which of the following actions will promote a safe transfer to the wheelchair? Obtain a mechanical lift Use a two-person assist Use a slide board Have the client stand and pivot

Obtain a mechanical lift Rationale: The Occupational Safety and Health Administration recommends a no-lift policy for all health care facilities. They advise using patient handling aids and mechanical lifting equipment for patients who are unable to assist in their transfer. The use of mechanical lifts and other patient handling devices reduces the risk of injury to nurses and clients. Patients who are unable to bear partial weight or full weight or who are uncooperative should be transferred using a full-body sling lift with two caregivers. This client is non-weight bearing and therefore, is a candidate for a lift and not for stand and pivot or any other means.

The nurse is planning care for a client who is paralyzed on the right side. Where should the nurse place the signaling device? On the left side of the bed near the client's hand Near the pillow on the right side Across the client's chest Attached to the bed

On the left side of the bed near the client's hand Any items the client needs to use must be place on the unaffected side. Placing them anywhere else or on the affected or paralyzed side will not allow the client to use them. Attaching the signaling device to the bed may be out of reach for the client.

The nurse has collected a stool specimen from a client with antibiotic associated diarrhea. Clostridium difficile is suspected. What action should the nurse take to transport the specimen to the lab? Place the specimen in a small biohazard bag. Wear gloves and an isolation gown when walking to the lab. Wipe the exterior of the collection cup with a disinfectant wipe. Place the client's label on the cap of the collection cup.

Place the specimen in a small biohazard bag. Rationale: Personal protection equipment (PPE) should be removed inside or just outside of the client's room. It is not worn in the hallway. Send or transport the specimen to the laboratory in a biohazard bag immediately or within the optimal time from collection as indicated by facility policy and guidelines. Avoid contact with soaps, detergents, and disinfectants as these may affect test results. The identification label should be attached to the cup so that when the lid is removed, the specimen remains labeled.

The nurse is caring for a client with severe osteoarthritis. Which of the following home interventions should the nurse recommend to promote client safety? Avoid the use of analgesics prior to activity. Perform exercise regimen at the end of the day. Use an assistive device for ambulation. Increase intake of high calorie foods.

Use an assistive device for ambulation. Rationale: Assistive devices allow a client with severe osteoarthritis to ambulate safely. Strenuous activity such as exercise should be planned for times when the client is in the least amount of pain, often this is in the morning hours. Analgesics are recommended prior to periods of significant activity. Increasing caloric intake does improve the safety of a client with osteoarthritis.

A nurse attended a training session on how to use a powered stand-assist lift to transfer a client to a chair. Which action by the nurse indicates correct use of the device? Places the sling under the client's arms Instructs the client to place the feet on the ground Holds the client's hands when standing Manually turns the handle to lift the client

Places the sling under the client's arms Rationale: Placing the sling under the client's arms is a correct method. The sling should be placed around the client's back and under the arms to support the torso during lifting. The feet should be placed on the footrests of the lift to support the weight during transfers. The nurse should instruct the client to place the hands on the lift handles. The powered stand-assist lift is designed to lift the client without assistance from the nurse.

A nurse is performing a bladder scan on a female client with history of a hysterectomy. Which action by the nurse indicates correct use of the ultrasound bladder scanner? Selects 'female' as the biological sex on the screen Applies ultrasound gel above the level of the umbilicus Points the directional icon on the scanner head toward the client's head Presses the scanner head upward toward the abdomen

Points the directional icon on the scanner head toward the client's head Rationale: The directional icon on the scanner head should be pointing toward the client's head to mimic the client's anatomical placement and obtain an accurate measurement. The nurse should select 'male' as the biological sex with clients who have a history of a hysterectomy due to the absence of the uterus. Ultrasound gel should be placed above the pubic symphysis, at the level of the bladder. The scanner head should be pointed down towards the coccyx to obtain an accurate read.

The nurse is adding sterile solution from an open container onto a prepared sterile field. Which action should the nurse take? Pour the sterile solution from a height of five inches Place the cap of the sterile solution on the table with edges down Pour the sterile solution immediately after opening the container Apply sterile gloves before opening the sterile solution container

Pour the sterile solution from a height of five inches Rationale: When adding sterile solution to prepared sterile field, the nurse should maintain the sterility of the solution and the field by pouring the solution 4 to 6 inches above the sterile container. When using a sterile solution container that has been opened, the nurse should place the cap with edges up and pour out solution lipping the bottle edges before pouring the solution onto the sterile field. The nurse will apply sterile gloves after pouring the sterile solution to prevent contamination.

The nurse is caring for a pediatric client who is experiencing a febrile seizure. Which action should the nurse take first? Administer anticonvulsant medication Protect the child's head from injury Loosen any clothing around the neck Apply a cooling blanket over the client

Protect the child's head from injury Rationale: Protecting the child from injury would be the highest priority action. Seizure activity may cause the child to have involuntary movements which could result in hitting their head. Loosening the clothing will help maintain the airway but would not be done first. The cooling blanket can help reduce the fever but would not be done first. Administering anticonvulsant medication would not happen first.

The nurse is participating in the planning of a facility's annual disaster drills. Which of the following indicates correct understanding of disaster planning? A mass casualty event involves management of the disaster by the facility using local resources. An internal disaster plan requires a strategy for evacuation and relocation of clients. A facility must plan disaster drills for all potential external natural disasters regardless of geographic location. A facility should plan active shooter drills primarily if gun violence is prevalent in the surrounding community.

Rationale: Facilities require annual drills for both internal and external disasters. Internal disaster plans always require a strategy for safely evacuating and relocating clients. A mass casualty event differs from a multi-casualty event in that it is broader in scope and typically overwhelms local medical capabilities, requiring collaboration with numerous healthcare facilities and possibly state, regional, and federal assistance. Facilities are required to plan drills for natural disasters but may choose disasters that are prevalent in their region, such as avalanches near mountains or hurricanes near the ocean. All facilities should plan active shooter drills, regardless of geographic location or prevalence of gun violence in the surrounding community

The nurse is providing education on prevention of repetitive stress injuries to an adult client who does virtual learning. Which of the following statements should be included in the teaching? "Lean back against the office chair while sitting at the computer." "Make the desktop higher than elbow height." "Use a mousepad with a wrist rest." "Float the wrists above the keyboard."

Rationale: Repetitive strain injury (RSI) is a general term used to describe the pain felt in muscles, nerves, and tendons caused by repetitive movement and overuse. The condition mostly affects the forearms and elbows, wrists and hands, and neck and shoulders. Leaning back against a desk chair can cause poor posture with hyperextension of the neck. Proper posture is essential to preventing RSI. The desktop should be at or below the level of the elbows to reduce shoulder strain. The wrists should be floating above all surfaces. A wrist rest can cause or worsen carpal tunnel syndrome.

The nurse is caring for a pediatric client a new diagnosis of cancer. The parent appears to be quite angry and has made threats towards staff members. Which of the following actions will reduce the risk of workplace violence while maintaining a caring environment? Call security to stand outside the client's room. Ignore the parent's behavior while providing care. Tell the parent that these feelings are inappropriate. Remain close to the doorway when talking with the parent.

Remain close to the doorway when talking with the parent. Rationale: Remaining close to the doorway allows the nurse to leave the room for assistance in case the parent becomes physically violent. While security may be notified of the potential need for their assistance, standing outside the room is likely to increase the anger of the parent. Ignoring the parent's behavior or telling them it is inappropriate does not validate the feelings that they are experiencing and decreases the nurse's awareness of safety concerns.

The nurse is caring for a client who is receiving a continuous infusion of norepinephrine. The infusion pump battery is not charging even though the pump is plugged in. Which of the following actions is appropriate? Continue to use the pump Replace it with another pump that has been charging properly. Inform the maintenance department that it will need to be serviced after the client is discharged Restart the pump

Replace it with another pump that has been charging properly. Rationale: The client is on a vasopressor to support hemodynamic stability. If the power were to fail or the client would need to leave the room, the pump would not continue to function and the client would not receive a life sustaining treatment. The pump should be replaced now while the situation isn't critical. The nurse should not attempt to troubleshoot software or mechanical failures.

A nurse is implementing fall precaution strategies for a client who is visually impaired. Which action would be an appropriate action for the nurse to take? Replace the call light with a touch pad call switch Place the bed in the lowest position Apply non-slip socks on the client Activate the position-sensitive bed alarm

Replace the call light with a touch pad call switch Rationale: Clients who are visually impaired are unable to see all of the buttons on the call light. Replacing the call light with a touchpad call switch enables the client to easily call for assistance. Placing the bed in the lowest position is a fall prevention strategy. However, this intervention benefits all clients regardless of impairment. Applying non-slip socks on the client will prevent a fall while ambulating. The nurse should perform actions specific to the client's impairment. Activating the position-sensitive bed alarm is a fall prevention strategy. However, bed alarms are beneficial for clients who have cognitive impairments and cannot understand instructions.

A nurse walks into a client's room and hears the unlicensed assistive personnel (UAP) telling the client, "If you continue to use that call bell for no reason, I am going to restrain you." What action does the nurse take? Reprimand the UAP in the client's room Instruct the UAP to make frequent rounds on the client Educate the UAP on the indication for restraints Report the UAP's statement to the unit manager

Report the UAP's statement to the unit manager Rationale: The statement made by the unlicensed assistive personnel (UAP) is considered assault. Threatening statements to a client should be reported to a supervisor. Reprimanding the UAP in the client's room is not professional behavior. Instructing the UAP to make frequent rounds on the client does not address the threatening statement. Educating the UAP on the use of restraints is indicated when medically necessary and does not address the threatening statement.

The nurse is walking a client to the bathroom who has a history of seizures. The client states "My vision seems to be distorted." What action should the nurse take first? Return the client to bed Take the client to the bathroom Assess the client's medication history Call the emergency response team

Return the client to bed Rationale: The patient is reporting signs and symptoms of an aura, which is a warning sign before a seizure. Returning the client to bed will reduce the likelihood of injury. Calling the response team and checking the history would not be the first actions. Continuing to the bathroom is not an appropriate action, as the client is experiencing difficulty with ambulation.

The nurse is teaching the parents of a toddler-age client about protective measures to reduce the risk of unintentional poisoning at home. Which of the following actions, if identified by the parent, indicates the need for further teaching? Attaching plastic hooks to cabinet doors Placing poison warning stickers on toxic substances Reusing empty containers to store different substances Storing cleaning agents and medications in locked cabinets

Reusing empty containers to store different substances Rationale: It requires further teaching if the parent reuses empty containers to store different substances. All substances should be kept in their original containers in the event of unintentional ingestion to alert the poison control center. It indicates a correct understanding of protective measures if the parent attaches plastic hooks to cabinets to keep them securely closed, stores medications and cleaning agents in locked cabinets, and places poison warning stickers on toxic substances.

A nurse is verifying a prescription for an intravenous antibiotic on the client's electronic medical record. Before administering the medication, which action will the nurse perform first? Disinfect the injection port on the intravenous line Program the infusion pump with the prescribed rate Flush the intravenous line with normal saline Scan the bar code on the client's wristband

Scan the bar code on the client's wristband Rationale: Client identification should occur before performing interventions. The client's bar code can be used as one identifier. Two identifiers are required. Disinfecting the injection port should be done after the client is identified. Programming the infusion pump is a step performed after the nurse verifies the client's identity. Flushing the intravenous line with normal saline is considered medication administration and cannot occur until the client has been properly identified.

The nurse is providing teaching about car seats to a client who is a first-time parent to a premature infant. Which information should the nurse include in the teaching? Use a convertible car seat with a covering Select a car seat with less than 5 ½ inch distance from the crotch strap to the seat back Choose a car seat that does not recline Pick a car seat that has head padding built into the head rest

Select a car seat with less than 5 ½ inch distance from the crotch strap to the seat back Rationale: An infant-only car seat should be used for preterm infants since convertible car seats are designed for full-term infants. Use a car seat with a distance of fewer than 5½ inches from the crotch strap to the seatback. This reduces the potential for your baby to slump forward. Infants should be in car seats that recline to keep the airway open. No head padding is recommended since the padding may push the infant's head forward.

A nurse is teaching a client with a hearing impairment about reporting adverse effects of intravenous therapy. Which action will promote client understanding of the instructions? Repeating the instructions to the client several times Sitting at eye-level in front of the client Speaking very loudly to the client Providing all instructions to the client in writing

Sitting at eye-level in front of the client Rationale: Clients with a hearing impairment benefit from lip reading and facial cues. Sitting close enough to the client will promote effective communication. Repeating the instructions several times may not guarantee understanding. Lack of client feedback may require rewording of the instructions. Speaking very loudly to the client does not promote effective communication. Clients with a hearing impairment benefit from visual cues, not a higher pitched voice. Providing all instructions to the client in writing does not promote therapeutic communication. The nurse should not assume the client is unable to communicate.

A staff nurse is assisting the unit charge nurse with narcotic counts. The staff nurse notices the charge nurse becomes nervous when several discrepancies in the counts are found. Which action should the staff nurse take? Stop the count and notify the unit manager. Continue the count and offer therapeutic statements to the charge nurse. Complete the count and document the discrepancies. Pause the count and find another staff nurse to assist the charge nurse.

Stop the count and notify the unit manager. Rationale: It is the nurses' responsibility to report narcotic discrepancies and behaviors indicative of substance abuse. The nurse should stop the count and notify a supervisor. Offering therapeutic statements to the charge nurse does not address the behavior change when discrepancies are found. Completing the count makes the staff nurse liable for the discrepancies. Pausing the count and finding another staff nurse does not address the issue of the charge nurse's reaction to the discrepancies.

The nurse is transporting a client in a motorized hospital bed when the cord becomes trapped under the wheel. The cord is visibly damaged. Which of the following actions is appropriate? Have the client remain in the unplugged bed until it is repaired Wrap the cord in electrical tape and continue to use the bed Exchange the bed and leave it in the hallway for repair Tag the bed for repair and remove it from the patient care area

Tag the bed for repair and remove it from the patient care area Rationale: Electrical equipment can present a safety hazard to both the patient and health care practitioner when safety measures are ignored. Electrical cords that are bent or twisted may have broken wires inside. Make certain that electric cords are not in a position to be trapped as beds are raised or lowered. This can strip insulation covering the electric wires. The bed should be removed from use and appropriately tagged for repair.

The nurse is caring for a client receiving an intravenous infusion using a smart pump when a system error begins alarming. Which action by the nurse will reduce the risk of injury to the client? Tag the device for maintenance and remove it from the unit Power down and restart the pump to clear the error Reposition the tubing in the infusion device to clear the alarm Place the plug in a red outlet to reduce the incidence of an energy surge

Tag the device for maintenance and remove it from the unit Rationale: Electrical equipment can present a safety hazard to both the patient and health care practitioner when safety measures are ignored. IV infusion equipment has the potential to experience software and system errors that can result in delayed infusions, over-infusion, under-infusion, or failed infusions. These errors can lead to serious injury and death. When system errors occur, the best action is to remove the device from service until it has been serviced. System errors are not caused by poor tubing placement and are not prevented by placing the plug in a generator outlet.

A nurse has administered 5 ml of intravenous hydralazine to a client. As the multi-dose medication vial is discarded, the nurse notes the prescription was to administer 2 ml. Which action does the nurse perform first? Inform the charge nurse Take the client's blood pressure Notify the healthcare provider Check the client's respirations

Take the client's blood pressure Rationale: Hydralazine is a vasodilator used in the treatment of hypertension. Upon noticing the dosage error, the nurse should immediately assess the client's condition by taking vital signs. Hydralazine decreases blood pressure. Informing the charge nurse and notifying the healthcare provider are important actions to manage the client's condition after the error. However, assessing the client is the priority. Checking the client's respirations will not evaluate the effects of hydralazine.

The nurse is assessing an older adult client for risk factors associated with falls. Which of the following should the nurse identify as increasing the risk of falls? Having a history of urinary tract infections Taking an opioid analgesic for pain Living in a home that has wall-to-wall carpeting Taking a daily anti-platelet medication

Taking an opioid analgesic for pain Rationale: Factors that increase the risk for falls in older adults include medication use such as opioid analgesics which can cause confusion and drowsiness, as well as diuretic use, which can cause orthostatic hypotension leading to dizziness. Antiplatelet medications are not associated with an increased fall risk, rather an increased bleeding risk. A history of a UTI will not increase the risk of falls, however, a current UTI with urinary frequency or urgency could increase an older adult client's risk of falls. Clients are at an increased risk of falls in houses with throw rugs, which could be tripping hazards.

The nurse is discharging a newborn from the hospital. Upon inspection of the car, which of the following is the correct use of the infant car seat? The car seat is positioned in the front seat. The car seat is forward facing in the back seat. The car seat is secured in the back seat by the door. The car seat is rear facing in the center of the back seat.

The car seat is rear facing in the center of the back seat. Rationale: The safest place for a car seat is the middle of the rear seat. Rear-facing seats are used for infants less than 20 lbs. Infants should never be placed in the front seat. Infant carriers are not the same as car seats.

The nurse is performing hand hygiene before providing care to a group of clients. The nurse should identify that the use of alcohol-based hand sanitizer would be contraindicated in which of the following clients? The client with a positive Methicillin-resistant Staphylococcus aureus (MRSA) infection The client receiving treatment for Clostridium difficile The client who has a history of Mycobacterium tuberculosis The client that developed a Escherichia coli urinary tract infection

The client receiving treatment for Clostridium difficile Rationale: Alcohol-based hand sanitizers are an alternate way to perform hand hygiene. However, the nurse should use soap and water when caring for clients with Clostridium difficile, which is a gram-positive, spore-forming bacteria that is not killed with alcohol. The nurse can use alcohol-based hand sanitizers with clients who have other nosocomial infections.

The nursing supervisor is working in a hospital that is in the path of a hurricane. Which client would be appropriate for immediate discharge? The client with nondisplaced tibia fracture that has been immobilized The client with lymphoma receiving induction IV chemotherapy The client with heart failure who is receiving 8 liters of oxygen The client who had an appendectomy with a paralytic ileus

The client with nondisplaced tibia fracture that has been immobilized Rationale: Medically unstable and unpredictable critical care patients are not candidates for discharge. Stable clients who need assistance are the second priority and, therefore, not discharged until the lowest priority clients are discharged. Ambulatory clients who need no assistance are the first clients to be safely discharged and relocated. The lowest acuity client here is the tibia fracture as this injury does not require surgical intervention. Clients who are receiving high flow oxygen, IV medication, and experiencing complications should not be discharged.

The nurse is assessing a client's room for safety hazards. Which finding observed by the nurse would increase the client's risk for falls? The client's side rails are raised on all sides of the bed. The client has a nonskid mat in the shower. The bed exit alarm is activated on the client's bed. The client's ambulatory aid is next to the client's bed.

The client's side rails are raised on all sides of the bed. Rationale: Raising all side rails on a bed has been shown to increase the risk of client falls because clients may become entrapped and unable to get out of bed. If a client is at risk for falls, side rails are often not used at all and beds are fully lowered, and padding is placed on the floor along the sides of the bed. Nonskid mats in the shower, bed exit alarms, and keeping client ambulatory aids within easy reach are all correct techniques to reduce the risk of falls.

A charge nurse is performing the daily check of the code cart on the unit. Which finding will the nurse report immediately for further inspection? The oxygen tank is empty The defibrillator charging light is off One of the wheels on the cart does not lock The last inspection is not signed

The defibrillator charging light is off Rationale: A defibrillator should always be fully charged in case of emergencies. Drained batteries can result in equipment failure. The oxygen tank is required for transport. However, the tank can be replaced with a full tank from the unit. The wheel locks prevent the crash cart from moving. Although the wheel needs to be inspected, it is not a priority action. Inspection signatures are important for quality improvement and documentation. However, ensuring the defibrillator is charged is the priority.

A nurse is performing an equipment check in a client's room. Which finding requires intervention? The sequential compression device sleeves inflate and deflate. The vital signs machine is connected to an outlet next to the window. The portable monitor charging indicator light is on. The excess infusion pump electrical cord is tied in a knot.

The excess infusion pump electrical cord is tied in a knot. Rationale: Electric cords should not be bent or twisted as this can cause the wires inside the cord to break. The nurse should loosen the cords. Outlets next to windows are not a safety hazard. The nurse should ensure electric cords are not near water sources. The charging light for heart monitors and defibrillators should be on, indicating that the batteries are charging. Sequential compression devices provide pressure to the leg in time increments. Inflation and deflation of the device is an expected finding.

A nurse is inspecting equipment in a client's restroom for safety. Which finding requires intervention by the nurse? A commode chair without the pan is in the shower area. The toilet has a raised seat with armrests. The nurse call pull cord is wrapped around the handrail. A rubber mat is spread out underneath the sink.

The nurse call pull cord is wrapped around the handrail. Rationale: Nurse-call pull cords in the shower should be hanging low enough for the client to pull in case of a fall. The nurse should unwrap the cord from the handrail. A commode chair provides support to a client with a physical disability or weakness while taking a shower. The pan is not necessary in the shower. A raised seat with armrests on the toilet provides support and decreases the risk of falls. A rubber mat prevents the client from slipping and falling on wet surfaces.

The charge nurse is observing a staff nurse perform a sterile dressing change for a client with sacral wound. Which action by the staff nurse while wearing sterile gloves would require the charge nurse to intervene? The nurse uses a sterile cotton-tipped swab to clean the wound edges. The nurse takes a sterile gauze pad and places it in the wound. The nurse picks up a gauze pad soaked in sterile saline to cleanse the wound. The nurse pulls up the clean sheet over the client's perineum for better draping

The nurse pulls up the clean sheet over the client's perineum for better draping Rationale: Touching the clean sheet with sterile gloves will contaminate the gloves. The draping should be completed before the sterile gloves are put on. Touching sterile objects with sterile gloves does not result in contamination.

A nurse attends a training on activation of the hospital incident command system (HICS). Which personnel role does the nurse identify as being responsible for rapidly evaluating clients to determine priorities for treatment? Public information officer Triage officer Hospital incident commander Charge nurse

Triage officer Rationale: A triage officer is responsible for evaluating clients and determining the priority for treatment. A public information officer is a person who serves as a liaison between the media and the hospital. The hospital incident commander is the person who assumes overall leadership when the emergency plan is implemented. The charge nurse is responsible for coordinating staff roles in the assigned unit.

The nurse is completing a home health visit for a client who uses a wheelchair. Which of the following findings indicate the need for home modification? Low countertops in the bathrooms A walk-in shower Upper cabinets in the kitchen are empty Uneven flooring in the home

Uneven flooring in the home Rationale: For the client who uses a wheelchair, having uneven flooring can be a significant safety issue and modifications are warranted. Lowered countertops and walk-in style showers are both appropriate. Not utilizing the upper cabinets in the home may be appropriate and is not a safety concern.

The nursing supervisor is working in an acute care facility following an earthquake. The building has lost water supply and is on generator power. Which patients should the nursing supervisor evacuate first? Ventilator dependent adults in the ICU Ambulatory adults on the medical unit Ambulatory children in the pediatric unit Non-ventilator dependent adults in the ICU

Ventilator dependent adults in the ICU Rationale: Evacuation decisions after No Advanced Warning Events such as earthquakes are based on building integrity, infrastructure, and environmental factors. If there is a potential or immediate threat to staff or clients, an assessment must be made to immediately evacuate or wait and reassess. Once evacuation is determined, triage is based on the availability of critical resources. In this case, the loss of power and water makes movement of acutely ill clients the priority. The other clients may be evacuated subsequently.

The nurse is working in the newborn nursery when an unfamiliar person in scrubs comes to the nursery door and requests to bring a newborn to the parents' room. What action by the nurse is appropriate? Verify the hospital identification badge Call security to the unit immediately Check with the parents to verify the request Ask the person if they are in the float pool

Verify the hospital identification badge Rationale: Each member of the hospital staff should have an identification badge clearly displayed. The nurse should look at the identification of anyone trying to transport a newborn as this is one way to prevent infant abduction. The individual may have legitimate reasons for being on the unit, so it is not appropriate to immediately call security, nor is it appropriate to take the person at their word.

The nurse is planning care for a client with a diagnosis of cancer who has received the first dose of intravenous chemotherapy. Which type of precautions should the nurse implement for this client? Place the client on contact isolation Wear a gown and gloves when handling linens and body fluids Place incontinence pads in the regular trash bin Maintain a distance of at least 3 feet from the client

Wear a gown and gloves when handling linens and body fluids Rationale: Chemotherapy drugs are present in the waste and body fluids of clients for 3 to 5 days after administration. Nurses involved in handling chemotherapeutic agents may be exposed to low doses of the agents by direct contact, inhalation, or ingestion. Therefore, PPE should be worn when handling the client's linens. Incontinence pads should be placed in chemotherapy waste bins. Distancing is required with radiation implants and not chemotherapy.


Kaugnay na mga set ng pag-aaral

Capstone Chap.6 Strategy Analysis and Choice

View Set

Milady Chapter 3 Advanced Esthetics Advanced Histology of the Cell and Skin

View Set

DECA Marketing Cluster Exam District Level Practice Test

View Set